Вы находитесь на странице: 1из 80

ndice General

1 Conjuntos 5
1.1 Descripcin de conjuntos . . . . . . . . . . . . . . . . . . . . . . . . . . . . . . 5
1.2 Conjunto de partes . . . . . . . . . . . . . . . . . . . . . . . . . . . . . . . . . 6
1.3 Relaciones entre conjuntos . . . . . . . . . . . . . . . . . . . . . . . . . . . . . 7
1.3.1 Inclusin de conjuntos . . . . . . . . . . . . . . . . . . . . . . . . . . . 7
1.3.2 Igualdad de conjuntos . . . . . . . . . . . . . . . . . . . . . . . . . . . 7
1.3.3 Diagramas de Venn . . . . . . . . . . . . . . . . . . . . . . . . . . . . . 7
1.4 Operaciones con conjuntos . . . . . . . . . . . . . . . . . . . . . . . . . . . . . 8
1.4.1 Unin de conjuntos . . . . . . . . . . . . . . . . . . . . . . . . . . . . . 8
1.4.2 Intersecccin de conjuntos . . . . . . . . . . . . . . . . . . . . . . . . . 8
1.4.3 Diferencia de conjuntos . . . . . . . . . . . . . . . . . . . . . . . . . . 9
1.5 Trabajo Prctico . . . . . . . . . . . . . . . . . . . . . . . . . . . . . . . . . . 11

2 Los nmeros reales 13


2.1 Los nmeros naturales . . . . . . . . . . . . . . . . . . . . . . . . . . . . . . . 13
2.1.1 Operaciones en el conjunto de los nmeros naturales . . . . . . . . . . 13
2.2 Los nmeros enteros . . . . . . . . . . . . . . . . . . . . . . . . . . . . . . . . 14
2.2.1 Orden en Z . . . . . . . . . . . . . . . . . . . . . . . . . . . . . . . . . 15
2.2.2 Valor absoluto . . . . . . . . . . . . . . . . . . . . . . . . . . . . . . . 15
2.2.3 Operaciones . . . . . . . . . . . . . . . . . . . . . . . . . . . . . . . . . 15
2.2.4 Jerarqua de las operaciones . . . . . . . . . . . . . . . . . . . . . . . . 17
2.3 Los nmeros racionales . . . . . . . . . . . . . . . . . . . . . . . . . . . . . . . 17
2.3.1 Propiedad de densidad . . . . . . . . . . . . . . . . . . . . . . . . . . . 18
2.3.2 Representacin de nmeros racionales . . . . . . . . . . . . . . . . . . 18
2.3.3 Operaciones con nmeros racionales . . . . . . . . . . . . . . . . . . . 19
2.3.4 Expresiones decimales peridicas infinitas . . . . . . . . . . . . . . . . 20
2.4 Los nmeros irracionales . . . . . . . . . . . . . . . . . . . . . . . . . . . . . . 21
2.5 Los nmeros reales . . . . . . . . . . . . . . . . . . . . . . . . . . . . . . . . . 22
2.5.1 Orden en R . . . . . . . . . . . . . . . . . . . . . . . . . . . . . . . . . 23
2.5.2 Notacin cientfica . . . . . . . . . . . . . . . . . . . . . . . . . . . . . 23
2.5.3 Diferencia y cociente de nmeros reales . . . . . . . . . . . . . . . . . 23
2.5.4 Potenciacin de nmeros reales . . . . . . . . . . . . . . . . . . . . . . 24
2.5.5 Propiedades de la potenciacin . . . . . . . . . . . . . . . . . . . . . . 24

1
2 NDICE GENERAL

2.5.6 Radicacin de nmeros reales . . . . . . . . . . . . . . . . . . . . . . . 25


2.5.7 Propiedades de la radicacin . . . . . . . . . . . . . . . . . . . . . . . 25
2.5.8 Exponentes racionales . . . . . . . . . . . . . . . . . . . . . . . . . . . 26
2.5.9 Racionalizacin de denominadores . . . . . . . . . . . . . . . . . . . . 26
2.6 Trabajo Prctico . . . . . . . . . . . . . . . . . . . . . . . . . . . . . . . . . . 27

3 Polinomios 31
3.1 Definicin . . . . . . . . . . . . . . . . . . . . . . . . . . . . . . . . . . . . . . 31
3.2 Grado . . . . . . . . . . . . . . . . . . . . . . . . . . . . . . . . . . . . . . . . 31
3.3 Valor numrico . . . . . . . . . . . . . . . . . . . . . . . . . . . . . . . . . . . 32
3.4 Igualdad . . . . . . . . . . . . . . . . . . . . . . . . . . . . . . . . . . . . . . . 32
3.5 Polinomios opuestos . . . . . . . . . . . . . . . . . . . . . . . . . . . . . . . . 32
3.6 Operaciones . . . . . . . . . . . . . . . . . . . . . . . . . . . . . . . . . . . . . 32
3.6.1 Suma . . . . . . . . . . . . . . . . . . . . . . . . . . . . . . . . . . . . 32
3.6.2 Resta . . . . . . . . . . . . . . . . . . . . . . . . . . . . . . . . . . . . 33
3.6.3 Producto . . . . . . . . . . . . . . . . . . . . . . . . . . . . . . . . . . 34
3.6.4 Divisin . . . . . . . . . . . . . . . . . . . . . . . . . . . . . . . . . . . 34
3.6.5 Raz de un polinomio . . . . . . . . . . . . . . . . . . . . . . . . . . . 36
3.6.6 Orden de multiplicidad de una raz . . . . . . . . . . . . . . . . . . . . 36
3.6.7 Teorema del factor . . . . . . . . . . . . . . . . . . . . . . . . . . . . . 37
3.7 Factorizacin de polinomios . . . . . . . . . . . . . . . . . . . . . . . . . . . . 37
3.7.1 Casos de factoreo . . . . . . . . . . . . . . . . . . . . . . . . . . . . . . 37
3.8 Trabajo Prctico . . . . . . . . . . . . . . . . . . . . . . . . . . . . . . . . . . 40

4 Ecuaciones 43
4.1 Las ecuaciones . . . . . . . . . . . . . . . . . . . . . . . . . . . . . . . . . . . 43
4.1.1 Introduccin . . . . . . . . . . . . . . . . . . . . . . . . . . . . . . . . 43
4.1.2 Dominio . . . . . . . . . . . . . . . . . . . . . . . . . . . . . . . . . . . 43
4.1.3 Conjunto solucin . . . . . . . . . . . . . . . . . . . . . . . . . . . . . 44
4.2 La ecuacin lineal . . . . . . . . . . . . . . . . . . . . . . . . . . . . . . . . . . 44
4.2.1 Ecuaciones equivalentes . . . . . . . . . . . . . . . . . . . . . . . . . . 45
4.2.2 Operaciones permitidas . . . . . . . . . . . . . . . . . . . . . . . . . . 45
4.2.3 Ejemplos . . . . . . . . . . . . . . . . . . . . . . . . . . . . . . . . . . 45
4.3 La ecuacin cuadrtica . . . . . . . . . . . . . . . . . . . . . . . . . . . . . . . 46
4.3.1 Ejemplos . . . . . . . . . . . . . . . . . . . . . . . . . . . . . . . . . . 47
4.3.2 Naturaleza de las races . . . . . . . . . . . . . . . . . . . . . . . . . . 48
4.3.3 Propiedades de las races . . . . . . . . . . . . . . . . . . . . . . . . . 50
4.3.4 Factorizacin . . . . . . . . . . . . . . . . . . . . . . . . . . . . . . . . 52
4.3.5 Ecuaciones de Primer Grado con dos incgnitas . . . . . . . . . . . . . 52
4.4 Sistemas de ecuaciones . . . . . . . . . . . . . . . . . . . . . . . . . . . . . . . 53
4.4.1 Interpretacin grfica . . . . . . . . . . . . . . . . . . . . . . . . . . . 56
4.5 Trabajo Prctico . . . . . . . . . . . . . . . . . . . . . . . . . . . . . . . . . . 57
NDICE GENERAL 3

5 Desigualdades 61
5.1 Introduccin . . . . . . . . . . . . . . . . . . . . . . . . . . . . . . . . . . . . . 61
5.2 Intervalos . . . . . . . . . . . . . . . . . . . . . . . . . . . . . . . . . . . . . . 62
5.2.1 Intervalo abierto . . . . . . . . . . . . . . . . . . . . . . . . . . . . . . 62
5.2.2 Intervalo cerrado . . . . . . . . . . . . . . . . . . . . . . . . . . . . . . 62
5.2.3 Intervalo semiabierto (o semicerrado) . . . . . . . . . . . . . . . . . . . 62
5.3 Desigualdades lineales . . . . . . . . . . . . . . . . . . . . . . . . . . . . . . . 64
5.4 Trabajo Prctico . . . . . . . . . . . . . . . . . . . . . . . . . . . . . . . . . . 66

6 Funciones 69
6.1 Introduccin . . . . . . . . . . . . . . . . . . . . . . . . . . . . . . . . . . . . . 69
6.2 Forma simblica de representar funciones . . . . . . . . . . . . . . . . . . . . 70
6.3 Definicin . . . . . . . . . . . . . . . . . . . . . . . . . . . . . . . . . . . . . . 71
6.4 Funciones lineales . . . . . . . . . . . . . . . . . . . . . . . . . . . . . . . . . . 71
6.5 Funciones cuadrticas . . . . . . . . . . . . . . . . . . . . . . . . . . . . . . . 72
6.6 Trabajo Prctico . . . . . . . . . . . . . . . . . . . . . . . . . . . . . . . . . . 75
4 NDICE GENERAL
Captulo 1

Conjuntos

Todos tenemos una idea de lo que es un conjunto, cuando decimos por ejemplo el conjunto
de animales del zoolgico, el conjunto de las provincias argentinas, el conjunto de letras del
abecedario.
Con respecto a este trmino, no daremos una definicin precisa. Slo diremos que un
conjunto es una coleccin de objetos que son los elementos del mismo.
En trminos matemticos conjunto es un concepto primitivo, es decir algo que se acepta
sin definicin.
Son ejemplos de conjuntos: el de estudiantes de la ciudad de Salta, el de presidentes de
la Repblica Argentina, etc.
Notacin: A los conjuntos se los denotan con letras maysculas de imprenta.
A los elementos de los denotan con letras minsculas.

1.1 Descripcin de conjuntos


Hay dos formas de describir un conjunto:

1. por extensin
2. por comprensin.

Para escribir un conjunto por extensin se nombran a sus elementos y se escriben entre
llaves y separados por coma.

Ejemplo 1 Sea A el conjunto formado por las vocales: A = {a, e, i, o, u}

Ejemplo 2 Sea B el conjunto de los primeros seis nmeros naturales B = {1, 2, 3, 4, 5, 6}

Esta forma de escribir a un conjunto es cmoda, a veces, cuando el mismo tiene pocos
elementos.
No importa el orden en que se escriben los elementos; los conjuntos son los mismos.
Adems los elementos se los escribe una sola vez.

5
6 CAPTULO 1. CONJUNTOS

Ejemplo 3 A = {a, e, i, o, u} es el mismo que A = {o, i, u, e, a} y tambin que A =


{u, e, o, i, a, u, o}

En este curso estudiaremos los conjuntos numricos.


Se denota con N al conjunto de los naturales:

N = {1, 2, 3, 4, . . .}

El uso de los puntos suspensivos es ambiguo, aunque en este caso todos entendemos que se
trata del conjunto de los nmeros naturales.
Algunas denominaciones para conjuntos:
N: conjunto de los nmeros naturales
Z: conjunto de los nmeros enteros
Q: conjunto de los nmeros racionales.
R: conjunto de los nmeros reales.
Para escribir un conjunto por comprensin, se da una propiedad que caracterice a sus
elementos. Es decir:
B = {x | p(x)}
Se lee el conjunto B est formado por los elementos x tales que cumplen la propiedad p(x).
La propiedad que se explicita debe ser tal que caracterice a esos elementos y solo a ellos.

Ejemplo 4 A = {x | x N}

Ejemplo 5 B = {x N | 5 x 9}

Hay conjuntos que tienen una cantidad finita de elementos, se los llama conjuntos fini-
tos(como el dado en el ltimo ejemplo) y otros que tienen una cantidad infinita de elementos,
son los conjuntos infinitos.
El conjunto vaco es el que no tiene elementos. Es un conjunto finito.
Notacin: Al conjunto vaco se lo denota con {} o bien .

Ejemplo 6 = {x | x es una hombre vivo que tiene trescientos aos}

El conjunto formado por todos los elementos del tema que se trate se llama conjunto
universal y se denota con U. Por ejemplo si trabajamos con nmeros, como los del ejemplo
el conjunto universal puede ser el conjunto de nmeros naturales. U = N.

1.2 Conjunto de partes


Dado un conjunto A el conjunto de partes es el conjunto formado por todos los subconjuntos
del conjunto dado. Se denota con P(A).

Ejemplo 7 Si A = {a, b, c}, halla P(A) y decide si las siguientes afirmaciones son verda-
deras o falsas:
1.3. RELACIONES ENTRE CONJUNTOS 7

(i) {a} P(A) (ii) {{b, c}} P(A) (iii) {} P(A) (iii) {b} P(A)
Solucin:
P(A) = {{a}; {b}{c}; {a, b}; {a, c}; {b, c}; {a, b, c}; }
(i) {a} P(A) F (ii) {{b, c}} P(A) V

(iii) {} P(A) F (iv) {b} P(A) V

1.3 Relaciones entre conjuntos


1.3.1 Inclusin de conjuntos
Se dice que un conjunto A est incluido en otro B si todo elemento del primero pertenece
al segundo. Se dice tambin que A es subconjunto de B, o que A est contenido en B.
Notacin: Se denota A B
Ejemplo 8 Sea A = {1, 2, 3} y B = {1, 2, 3, 5, 6}. Se cumple que A B ya que 1 B,
2 B, 3 B.

1.3.2 Igualdad de conjuntos


Dos conjuntos son iguales si tienen los mismos elementos, es decir si todo elemento de A es
tambin elemento de B y si todo elemento de B es elemento de A.
En smbolos:
A = B si A B y B A
Si A es un subconjunto de B, pero A y B no son iguales, se dice que A es un subconjunto
propio de B.
Ejemplo 9 Sea A = {0, 2} y B = {x | x2 2x = 0}. Se cumple que A = B

1.3.3 Diagramas de Venn


Hay una manera de representar los conjuntos, usando los diagramas de Venn. Para ello se
usa una curva cerrada en cuya regin interior se colocan los elementos que pertenecen al
conjunto. En general para representar al conjunto universal se utiliza una regin cerrada en
forma de rectngulo.

Los smbolos pertenencia e inclusin


El smbolo se utiliza para indicar que un elemento est en el conjunto y el smbolo se
utiliza para indicar que un conjunto est incluido en otro.
Ejemplo 10 Sean A = {2, 3, 4, 5, 6} B = {4, 5, 6}
Se observa que:
(a) 2 A, 3 A, 4 A, 5 A, 6 A
(b) Tambin que B A ya que todo elemento de B pertenece a A.
8 CAPTULO 1. CONJUNTOS

1.4 Operaciones con conjuntos


1.4.1 Unin de conjuntos
Dados dos conjuntos A y B se llama unin de A y B al conjunto formado por los elementos
que pertenecen a A o a B.
Notacin: La unin de dos conjuntos se denota con A B.
En smbolos A B = {x | x A x B}
Nota: El smbolo significa o lgico, se toma en sentido incluyente; significa que los
elementos que estn en la unin estn en A, en B o en ambos conjuntos.
Grficamente:

Ejemplo 11 Sean A = {x | x N x < 5} B = {x | x N 3 x < 8}

A B = {1, 2, 3, 4, 5, 6, 7}

Valen las siguientes leyes de la unin de conjuntos:

(a) A B = B A (conmutativa)
(b) (A B) C = A (B C) (asociativa)
(c) A = A (existencia de elemento neutro)

1.4.2 Intersecccin de conjuntos


Dados dos conjuntos A y B se llama interseccin de A y B al conjunto formado por los
elementos comunes a los dos conjuntos.
Notacin: La interseccin de conjuntos se denota con A B.
En smbolos:
A B = {x | x A x B}

Nota: El smbolo significa y lgico y significa que los elementos que estn en la interseccin
estn en A, y estn en B.
Grficamente:
1.4. OPERACIONES CON CONJUNTOS 9

Cuando dos conjuntos no tienen ningn elemento en comn se dicen disjuntos. En este
caso la interseccin es el conjunto vaco.

Ejemplo 12 Sea A = {2, 4, 6, 8} B = {4, 6, 8, 10}

A B = {4, 6, 8}

Ejemplo 13 Sea A = {2, 4, 6, 8} B = {1, 3, 5, 7}

AB =

En este caso los conjuntos A y B son disjuntos.


Valen las siguientes leyes de la interseccin de conjuntos:

(a) A B = B A (conmutativa)
(b) (A B) C = A (B C) (asociativa)
(c) A = (existencia de elemento neutro)

1.4.3 Diferencia de conjuntos


Dados dos conjuntos A y B se llama diferencia entre A y B al conjunto formado por todos
los elementos que pertenecen a A, pero no pertenecen a B.
Notacin: La diferencia entre A y B se denota con A B.
En smbolos:
A B = {x | x A x / B}
Grficamente:
10 CAPTULO 1. CONJUNTOS

El complemento de un conjunto que se denota con A es el conjunto U A


En smbolos:
U A = {x | x U x
/ A}

Ejemplo 14 Sean los conjuntos A = {x | x N x < 8}


B = {x | x N x 10}

Escribe:
(a) A B (b) BA
Solucin:

(a) A B =
(b) B A = {8, 9, 10}

Valen las siguientes propiedades:


(A B) = A B 
(A B) = A B 

Ejemplo 15 Para los conjuntos dados: U ={x Z | 6 x 6}; A = {x Z |x es


impar 6 < x < 6};

B = {6, 2, 0, 2, 4, 6}; C = {5, 1, 1, 2, 3, 4}, realiza las siguientes operaciones (B C


A) C
Solucin:
Escribimos a los conjuntos U y A por extensin:
U ={x Z | 6 x 6} = {6, 5, 4, 3, 2, 1, 0, 1, 2, 3, 4, 5, 6}

A = {x Z |x es impar 6 < x < 6} = {5, 3, 1, 1, 3, 5}

B = {6, 2, 0, 2, 4, 6} C = {5, 1, 1, 2, 3, 4}

Hallamos B C y la diferencia de B C con A

B C = {5, 4, 3, 1, 1, 3, 5} (B C A) = {4}

Finalmente la operacin pedida:

(B C A) C = {4} {5, 1, 1, 2, 3, 4} = {5, 4, 1, 1, 2, 3, 4}

Ejemplo 16 Dados los conjuntos A, B, C, indica qu operacin representa la zona som-


breada:

Solucin:
(A C) B
1.5. TRABAJO PRCTICO 11

Figura~1.1:

1.5 Trabajo Prctico


1. Define por comprensin o extensin segn corresponda:

(a) A = {x | x es un planeta del sistema solar}


(b) B = {Lunes, Martes, Mircoles, Jueves, Viernes, Sbado, Domingo}
(c) C = {Primavera, Verano, Otoo, Invierno}
(d) D = {x | x es mltiplo de 5 comprendido entre 13 y 42}
(e) E = {x | x es un nmero natural menor que 6}
(f) F = {3, 4, 5, 6, 7, 8, 9, 10}
(g) G = {3, 6, 9, 12, 15, 18, ...}
(h) H = {x | x es un nmero impar}

2. Cules de los siguientes conjuntos son: vacos, unitarios, finitos, infinitos?

(a) A = {x | x es da de la semana}
(b) B = { vocales de la palabra conjunto}
(c) C = {1, 3, 5, 7, 9, .....}
(d) D = {x | xes un nmero par}
(e) E = {x | x < 15}
(f) F = {x | es la solucin de f (x) = |x|}

3. Decide si A B o B A o ninguna de las dos, segn corresponda.

(a) A = {x | x es un nmero natural}


B = {x | x es un nmero par}
12 CAPTULO 1. CONJUNTOS

(b) A = {x | x es un mltiplo de 3}
B = {x | x es un mltiplo de 6}
(c) A = {x | x es un nmero natural comprendido entre 2 y 5}
B = {3, 4}
(d) A = {x | x es un nmero de la forma 2k, con k N}
B = {x | x es un nmero de la forma 2k + 1, con k N}

4. Coloca el signo ; ;
/ ;  ; segn corresponda:
a) 2 {x | x es un nmero primo}
b) 2 {1, {2} , 2}
c) {{2}} {1, {2} , 2}
d) {{1, 2}} {1, {2} , {2, 3}}
e) 4 {x | x es divisor de 18}
f) {x | x es divisor de 12} {x | x es mltiplo de 2}
5. Calcula la unin entre A y B :

(a) A = {x | /x es un nmero natural} B = {x | x es un nmero impar}


(b) A = {x | x es un tringulo equiltero} B = {x | x es un tringulo issceles}

6. Calcula la interseccin entre A y B:

(a) A = {x | x es un nmero natural} B = {x | x es un nmero par}


(b) A = {x | x es un tringulo issceles} B = {x | x es un tringulo rectngulo}

7. Sean U = {1, 2, 3, 4, 5, 6, 7, 8, 9} ; A = {1, 2, 3, 4}; B = {4, 5, 6, 7}; C = {5, 6, 7, 8, 9}


Halla
(a) A B (b) A (c) (A B) (A C)
(d) A B (e) B (f) C A
(g) A (B C) (h) (A B) (B A) (i) B B
8. Dado el conjunto A = {6, 2, 8, 4, 3} encuentra todos los subconjuntos de A que se
puedan construir con sus elementos.
9. Sean los conjuntos A, B, y C no vacos, usa diagramas de Venn para ilustrar los
resultados obtenidos al efectuar las operaciones indicadas en las expresiones dadas:
(a) AB (b) AC (c) AB
 
(d) BA (e) ABAC (f) AB C
   
(g) AB BA C
Captulo 2

Los nmeros reales

2.1 Los nmeros naturales


Los primeros nmeros que aparecieron, histricamente, fueron los nmeros naturales: 1, 2, 3, 4, . . .
que sirven para contar. Se representan en la recta numrica mediante puntos, fijando un
origen, una unidad y una escala.

Al conjunto de los nmeros naturales se lo simboliza con N:


N = {1, 2, 3, 4, . . .}
En el conjunto de los nmeros naturales pueden identificarse dos conjuntos: el de los pares
y el de los impares. El conjunto de los pares est formado por los nmeros 2, 4, 6, 8, . . . y el
de los impares por 1, 3, 5, 7, 9, . . .. Se observa que un nmero natural par puede expresarse
como a = 2k con k N. Por ejemplo 10 = 2 5.
Si un nmero no es par es impar. La expresin simblica para un nmero impar es
a = 2k + 1, con k N.

2.1.1 Operaciones en el conjunto de los nmeros naturales


En N estn definidas las operaciones de suma y producto. Si sumamos o multiplicamos dos
nmeros naturales el resultado es siempre un nmero natural, lo que nos dice que el conjunto
de los naturales es un conjunto cerrado para la suma y para el producto.
En N valen las propiedades conmutativa y asociativa -con respecto a estas dos operaciones-
y la propiedad distributiva del producto con respecto a la suma. Por ejemplo:
(conmutativa de la suma
2+5 =5+2 25=52
y del producto)

13
14 CAPTULO 2. LOS NMEROS REALES

Es decir, la propiedad conmutativa establece que la suma es la misma independientemente


del orden en que se efecte la adicin. La misma propiedad vale para el producto: el producto
de dos nmeros es independiente del orden de los factores.

(asociativa de la suma
(3 + 2) + 5 = 3 + (2 + 5) (3 2) 5 = 3 (2 5)
y del producto)

Para tres o ms nmeros dados, la suma es la misma, independientemente del orden en el


cual los nmeros se adicionen. Igual para el producto: el producto es el mismo, indepen-
dientemente del orden en el cual los nmeros se multipliquen.

(distributiva del producto


2 (3 + 4) = 2 3 + 2 4 (3 + 4) 5 = 3 5 + 4 5
respecto de la suma)

Con respecto a la diferencia, podemos decir que si restamos dos nmeros naturales, no
siempre obtenemos por resultado un nmero natural; por ejemplo, podemos restar 104 = 6
y obtenemos un nmero natural (se expresa, en smbolos, 6 N, y se lee 6 pertenece a N).
Pero si queremos restar, por ejemplo, 9 12, el resultado no es un nmero natural (en
smbolos, 9 12 / N).
Surge as la necesidad de ampliar el conjunto de los nmeros naturales -con el cero y
los enteros negativos- formndose as el conjunto de los nmeros enteros.
Los nmeros enteros negativos son tiles para expresar ciertas temperaturas, algunas
profundidades, prdidas, etc. Por ejemplo, si en cierto lugar hacen 10 C bajo cero, puede
decirse que la temperatura es de 10 C; una profundidad de 7 m bajo el nivel del mar se
escribe como de 7 m.

2.2 Los nmeros enteros


Los nmeros naturales (tambin llamados enteros positivos), el cero y los enteros nega-
tivos, forman el conjunto de los nmeros enteros, que se simboliza con Z:

Z = {. . . , 4, 3, 2, 1, 0, 1, 2, 3 . . .}

Si nos situamos sobre la recta numrica, podemos asociar con cada nmero natural otro
nmero que est a la misma distancia del 0 pero del lado opuesto. Nmeros como 4 y 4,
que estn a la misma distancia del cero, se llaman opuestos. Se cumple que la suma de
nmeros opuestos da por resultado 0, que es el elemento neutro para la suma.
La representacin en la recta numrica de los nmeros enteros se realiza mediante puntos:
los enteros positivos estn a la derecha del cero y los enteros negativos a la izquierda de cero.
El cero no es positivo ni negativo.
2.2. LOS NMEROS ENTEROS 15

2.2.1 Orden en Z
Dados dos nmeros enteros a y b, se dice que a es menor que b si a est a la izquierda de
b en la recta numrica; esta relacin se denota con a < b.

Ejemplo 17

(a) 5 < 9, ya que 5 est a la izquierda de 9.


(b) 10 < 3, ya que 10 est a la izquierda de 3 sobre la recta numrica.

Ejercicio 2.2.1 Cuntos nmeros enteros hay entre dos enteros distintos a y b?

2.2.2 Valor absoluto


El valor absoluto de un nmero entero es su distancia al cero sobre la recta numrica.
Usamos el signo | x | para representar el valor absoluto del nmero x. El valor absoluto de
un nmero es siempre positivo o cero.

Ejemplo 18 El valor absoluto de 4 es su distancia al cero, que es 4; el valor absoluto de


4, su distancia al cero, tambin es 4. Se escribe | 4 |=| 4 |= 4:

Definicin 19 Para cualquier nmero entero x, su valor absoluto se define como:



x si x 0
| x |=

x si x < 0

Nota: x significa el opuesto de x y no necesariamente representa un nmero negativo.

2.2.3 Operaciones
Los nmeros usados para representar puntos, deudas o identificar jugadores de ftbol, tienen
significacin. Pero lo que realmente los hace tiles es nuestra aptitud para combinarlos, que
es sumarlos, restarlos y multiplicarlos.
La resta de nmeros enteros a y b se define como la suma entre a y el opuesto de b. Es
decir:
a b = a + (b)
16 CAPTULO 2. LOS NMEROS REALES

Ejemplo 20 9 12 = 9 + (12) = 3
Se puede pensar en esta suma utilizando la recta numrica. Para sumar 9 con 12,
comenzamos con 9 y nos movemos 12 hacia la izquierda; as llegamos a 3.
En la prctica, para sumar nmeros enteros se usan las siguientes reglas:
Regla 1. (Suma de dos nmeros positivos o dos enteros negativos)

Se suman los valores absolutos y el resultado tiene el mismo signo que los sumandos.
Regla 2. (Suma de un nmero positivo y uno negativo)
Se restan los valores absolutos, el del mayor menos el del menor.
El resultado tiene el signo del sumando con mayor valor absoluto.
1. (a) 5 + 7 = 2
(b) 4 + 3 2 5 + 9 = (4 + 3) 2 5 + 9 = 1 2 5 + 9 =
= (1 2) 5 + 9 = 3 5 + 9 = (3 5) + 9 = 8 + 9 = 1
O tambin:
4 + 3 2 5 + 9 = 4 2 5 + 3 + 9 = (4 2 5) + (3 + 9) =
= 11 + 12 = 1
Nota: cuando hay varios sumandos, se suman todos los positivos y se suman todos los
negativos y se aplica la regla 2.
As, en el ltimo ejemplo,
4 + 3 2 5 + 9 = (3 + 9) + (4 2 5) = 12 11 = 1
Para el producto de nmeros enteros, se aplica la regla de los signos, por todos conocida:
(a) (b) = a (b) = (ab)

(a) (b) = a b = ab
Esto nos dice que el producto de dos nmeros enteros de distintos signos es negativo. El
producto de dos enteros de igual signo es positivo. (cuando ambos son positivos o ambos
negativos).
1. (a) (3) 7 = 21
(b) (2) (5) = 10
Con los nmeros enteros no siempre podemos realizar la divisin. Por ejemplo 8 4 = 2
y 2 Z; pero si queremos dividir 7 entre 3, vemos que 7 3 / Z. Para que la divisin en
Z sea posible, el divisor debe estar contenido un nmero exacto de veces en el dividendo, lo
que se expresa diciendo que el dividendo es mltiplo del divisor (en el ejemplo dado, 8 es
mltiplo de 4 ya que 8 = 4 2). En general, dados dos enteros a y b, se dice que a es mltiplo
de b si a = k b, con k nmero entero.
1. (a) 12 es mltiplo de 4 ya que 12 = 3 4
(b) 18 es mltiplo de 2 ya que 18 = 2 (9)
2.3. LOS NMEROS RACIONALES 17

2.2.4 Jerarqua de las operaciones


Los parntesis son signos de agrupacin. Cuando una expresin contiene parntesis se de-
ben resolver primero las operaciones que estn dentro de ellos. Se realiza las operaciones
siguiendo el siguiente orden de jerarqua:

1. Efectuar los clculos dentro de los signos de agrupacin.


2. Mutiplicar y dividir de izquierda a derecha.
3. Sumar y restar de izquierda a derecha.

2(3)+25(5)+(3+54)+2(5+6+4) = 65+(2)+25 = 652+10 = 3

2.3 Los nmeros racionales


Se ampla el conjunto Z incorporando las fracciones, para que siempre sea posible dividir
enteros, es decir para que expresiones de la forma a x = b, donde a y b son enteros y a = 0,
tengan siempre solucin. Por ejemplo, la expresin 2 x = 5 tiene solucin y sta se expresa
5
como x = , que es un nmero fraccionario.
2
El conjunto de los enteros, con el conjunto de las fracciones forman el conjunto de los
nmeros racionales, que se denota con Q:
a
Q= tal que a y b Z y b = 0
b
a
Es decir, cualquier nmero que se pueda expresar como un cociente de dos enteros ,
b
con b = 0, es un nmero racional.
1 3 17
Ejemplo 21 Son nmeros racionales , , , 5 y 0.
5 7 4
3 6 15
Las fracciones , , , . . ., todas expresiones del mismo nmero racional, se dice que
4 8 20
son equivalentes. Entre esas infinitas fracciones equivalentes, se utiliza para representar al
3
nmero racional a la fraccin , la que se denomina forma reducida. Entonces, la forma
4
reducida de un nmero racional es la fraccin obtenida luego de realizar todas las simplifi-
caciones posibles entre el numerador y el denominador y -por convencin- con denominador
positivo.
a c
En general, se dice que dos nmeros racionales y son equivalentes si a d = b c y
b d
se expresa:
a c
= ad =bc (con b = 0 y d = 0)
b d
2 4 8 2 2
As, por ejemplo, = = = . . .. = . . .. En este caso, es la forma reducida y
5 10 20 5 5
representa a todas las fracciones equivalentes a ella.
18 CAPTULO 2. LOS NMEROS REALES

2.3.1 Propiedad de densidad


Una propiedad importante del conjunto de los nmeros racionales es la densidad. Se dice
que el conjunto Q es denso, lo que significa que entre dos racionales distintos siempre existe
otro racional, y por lo tanto siempre existen infinitos nmeros racionales comprendidos entre
dos cualesquiera de ellos.
Para encontrar un nmero racional entre dos racionales distintos, primero los ordenamos
de menor a mayor y luego hallamos la semisuma de ellos. 1 El nmero as obtenido se
encuentra entre los dos dados. Se puede aplicar idntico procedimiento entre uno de los
nmeros dados y el nmero racional hallado; as, repitiendo este proceso, se encuentra cada
vez otro nmero racional, que est comprendido entre los dados.

2 5
Ejercicio 2.3.1 Halla un nmero racional entre y .(Sugerencia: transforma ambos
7 3
nmeros en sus equivalentes de igual denominador).

2.3.2 Representacin de nmeros racionales


A cada nmero racional le corresponde un punto un punto sobre la recta numrica. Cuando
dibujamos el punto correspondiente a un nmero sobre la recta nmerica, decimos que hemos
graficado el nmero.

3 7
Ejemplo 22 Representa sobre la recta nmrica: (a) (b) .
5 4
(a) Se divide la unidad de la recta numrica en quintos:

1
A partir del 0, se marca -a la derecha- 3 veces un segmento de medida igual a .
5
La ltima marca representa al nmero.
(b) Se divide la unidad de la recta numrica en cuartos:

A partir del 0, se marca -a la izquierda- 7 veces un segmento de medida igual a


1
. La ltima marca representa al nmero.
4
1 En el apartado veremos cmo se suman racionales
2.3. LOS NMEROS RACIONALES 19

Si un nmero racional tiene el denominador mayor que el numerador, su representacin


en la recta est entre 0 y 1. Si el numerador es mayor que el denominador, el nmero equivale
a varias unidades completas ms una fraccin de unidad.
7 3
1. (a) =1+
4 4
19 5
(b) =2+
7 7
Si nos preguntamos si con los nmeros racionales se cubre por completo la recta numrica,
la respuesta es no, ya que existen infinitos puntos en la recta a los que no les corresponde un
nmero racional. Luego veremos que estos puntos representan a los nmeros irracionales.

2.3.3 Operaciones con nmeros racionales


Suma y resta
Para sumar o restar dos fracciones de igual denominador, se suman los numeradores y se
coloca el mismo denominador.
3 7 10
Ejemplo 23 + = =2
5 5 5
Si las fracciones no tienen el mismo denominador, se reemplazan por otras fracciones
equivalentes y que tengan el mismo denominador. Por lo tanto el mtodo consiste en trans-
formar las fracciones, en fracciones de denominador comn.
3 2 4 45 24 80 11
Ejemplo 24 + = + =
4 5 3 60 60 60 60

Producto
El producto de dos nmeros racionales es otro nmero racional, cuyo numerador es el pro-
ducto de los numeradores y el denominador es el producto de los denominadores de las
fracciones dadas.
4 7 47 28 7
Ejemplo 25 = = =
3 20 3 20 60 15
Observacin: en la prctica, conviene simplificar, en lo posible, numerador con denomi-
nador.

Cociente
Para dividir dos racionales se multiplica el primero por el inverso multiplicativo del segundo.
a b a b
Observacin: si el nmero es , su inverso multiplicativo es . Se cumple que = 1
b a b a
y 1 es el elemento neutro con respecto a la multiplicacin.
3 2 3 7 21
Ejemplo 26 = =
4 7 4 2 8
20 CAPTULO 2. LOS NMEROS REALES

2.3.4 Expresiones decimales peridicas infinitas


Un nmero racional puede ser expresado como una expresin decimal peridica infinita. Por
1 1
ejemplo: = 0, 250000 . . . ; = 0, 33333 . . ..
4 3
Una expresin decimal peridica infinita es un nmero decimal que tiene una parte
entera y una parte decimal que tiene infinitas cifras decimales peridicas (que se repiten).
En el primer ejemplo, la parte peridica est formada por el cero, por lo puede omitirse
1
su escritura, resultando entonces = 0, 25. Esta expresin se dice tambin que es una
4
expresin decimal finita.
Las fracciones equivalentes a aqullas que tienen de denominador una potencia de diez,
2 4
resultan ser expresiones decimales finitas. Por ejemplo = = 0, 4.
5 10
Las fracciones que no son equivalentes a aqullas cuyo denominador es una potencia de
7
diez, resultan ser expresiones decimales peridicas. Por ejemplo: = 2, 333 . . . .
3
Las expresiones decimales peridicas se clasifican en peridicas puras y peridicas mixtas.
En las peridicas puras, el perodo est inmediatamente despus de la parte entera, por
ejemplo 0, 33333 . . . que se expresa como 0,

3 y es una expresin peridica pura con perodo


3.
En las peridicas mixtas, luego de la parte entera aparecen cifras no peridicas seguidas
del perodo; por ejemplo 1, 325656565656 . . ., que se escribe como 1, 32 56.
a
Para expresar un nmero racional como una expresin decimal peridica, se realiza la
b
divisin entre el numerador a y el denominador b.

Ejemplo 27

2
(a) = 0, 2222 . . . = 0,

2
9
3
(b) = 0, 60000 . . . = 0, 6
5
113
(c) = 1, 255555 . . . = 1, 2

5
90

Cmo se realiza el procedimiento inverso? Es decir, dada la expresin decimal de un


nmero racional, cmo se la escribe como un cociente de dos enteros? (Recordemos las
reglas estudiadas en el colegio.)

Ejemplo 28

3 1
(a) 0,

3= =
9 3
124 12 112 28
(b) 0, 12

4= = =
900 900 225
2.4. LOS NMEROS IRRACIONALES 21

Porcentaje

Los porcentajes se utilizan, muchas veces, para describir los incrementos o disminuciones en
cantidades como precios, poblacin, salarios, etc.
Los nmeros decimales, algunas veces se expresan como porcentajes, por ejemplo 7%
7
significa o 0, 07.
100
Una forma simple de convertir un nmero decimal a porcentaje, es multiplicar el decimal
100
por el nmero 1, escrito como el 100% (100% = ). Por ejemplo:
100

0, 54 = 0, 54 1 = 0, 54 100% = 54%

2.4 Los nmeros irracionales


Como ya se expres anteriormente, existen
puntos en la recta a los que no les corresponde
ningn nmero racional; por ejemplo 2. A este nmero no se lo puede expresar como el
cociente de dos enteros. Este es un ejemplo de unnmeroirracional.

Otros nmeros irracionales son, por ejemplo, 3, 5, 11, .
Los nmeros irracionales son expresiones decimales infinitas no peridicas; al conjunto
de los nmeros irracionales se lo representa con I.

Ejercicio 2.4.1 Observa el siguiente tringulo rectngulo construido sobre la recta numrica
y cuyos catetos tienen una unidad de longitud:

(a) Calcula el valor de la hipotenusa.


(b) Utiliza el comps y, con centro en el punto 0, construye un arco de radio igual a
la hipotenusa que intersecte a la recta numrica. Qu nmero representa dicho
punto sobre la recta numrica.

La unin del conjunto de los nmeros racionales con el de los irracionales forma el con-
junto de los nmeros reales. A ste se lo denota con R.

R=QI

Los nmeros reales s cubren toda la recta numrica.


En el siguiente esquema se nombran los conjuntos numricos estudiados y cmo ellos se
relacionan:
22 CAPTULO 2. LOS NMEROS REALES

Nmeros naturales










Nmeros


Cero



enteros


Nmeros



Enteros negativos
racionales



Nmeros reales



Nmeros




fraccionarios







Nmeros


irracionales

2.5 Los nmeros reales


Las propiedades que se aceptan sin demostrar se llaman axiomas. En los nmeros reales se
consideran vlidos los siguientes axiomas de la igualdad.

Reflexividad: a = a.
Simetra: si a = b, entonces b = a.
Transitividad: si a = b y b = c, entonces a = c.

Estn definidas dos operaciones, suma y producto, y se verifican los siguientes axiomas:
Con respecto a la suma

Ley de cierre: si a R y b R, entonces a + b R.


Ley conmutativa: si a R y b R, entonces a + b = b + a.
Ley asociativa: a + (b + c) = (a + b) + c, cualesquiera sean los nmeros reales a, b y c.
Existencia de elemento neutro: existe el nmero real 0, tal que a + 0 = 0 + a = a,
cualquiera sea el nmero real a.
Existencia de opuesto o inverso aditivo: cualquiera sea el nmero real a, existe un
nico nmero real a tal que se cumple que: a + (a) = a + a = 0.

Con respecto a la multiplicacin

Ley de cierre: si a R y b R, entonces a b R.


Ley conmutativa: si a R y b R, entonces a b = b a.
Ley asociativa: a (b c) = (a b) c, cualesquiera sean los nmeros reales a, b y c.
Existencia de elemento neutro: existe el nmero real 1, tal que a 1 = 1 a = a,
cualquiera sea el nmero real a.
2.5. LOS NMEROS REALES 23

Existencia de inverso multiplicativo: cualquiera sea el nmero real a, distinto de cero,


existe un nico nmero real que se denota con a1 tal que: a a1 = a1 a = 1.

Una propiedad que vincula a las operaciones de suma y producto es la llamada ley
distributiva que simblicamente expresa: a (b + c) = a b + a c, cualesquiera sean los
nmeros reales a, b y c.
Los nmeros naturales y los nmeros enteros estn incluidos en los reales; sin embargo
para ellos no se cumplen algunas propiedades de los nmeros reales.
Cules de stas propiedades no se cumplen en Z? Cules no se verifican en Q?

2.5.1 Orden en R
En R est definida una relacin de orden, la relacin menor que. Se dice que a < b si se
verifica que b a > 0. Esta relacin tambin vale si en lugar de menor se tiene menor o
igual que.

Ejemplo 29 1 < 2 ya que 2 1 > 0.

Observemos que 1 se encuentra a la izquierda de 2 en la recta.

2.5.2 Notacin cientfica


La notacin cientfica es til cuando se trabaja con nmeros muy grandes o muy pequeos.
Cualquier nmero real positivo puede escribirse en la forma a 10n donde 1 a < 10 y
n es un nmero entero. Decimos que un nmero as escrito est expresado en notacin
cientfica.

Ejemplo 30

(a) 300.000.000.000.000 = 3 1014


(b) 0, 00000000000261 = 2, 61 1012

2.5.3 Diferencia y cociente de nmeros reales


Dados dos nmeros reales a, b, la diferencia a b se define como:

a b = a + (b)

donde (b) es el opuesto de b.


Si b = 0, el cociente se define como:
a
= a b1
b
donde b1 es el inverso de b.
24 CAPTULO 2. LOS NMEROS REALES

2.5.4 Potenciacin de nmeros reales


Sea a un nmero real cualquiera y n un entero positivo. Entonces definimos an como:

an =a
 a a . . . a
n veces

As, a3 = a a a y a5 = a a a a a.
El nmero a se llama base, n es el exponente y an la potencia ensima de a.
Es por todos conocida la regla de los signos siguiente: Si la base es positiva el resultado
de la potencia es positivo.
La potencias de exponente par, con base negativa tienen siempre resultado positivo. Si
la base es negativa y el exponente es impar el resultado de la potencia es negativo.
 2
2 4
Ejemplo 31 (2) = 16 4 3
(3) = 27 =
5 25

Sobre la base de la definicin, obtenemos las leyes de los exponentes. Siendo a, b R,


m y n enteros positivos (en la propiedad 3 supondremos que m > n), valen las siguientes
propiedades.

2.5.5 Propiedades de la potenciacin


1. am an = am+n
2. (am )n = amn
am
3. = amn (a = 0)
an
4. (a b)n = an bn
 a n an
5. = n (b = 0)
b b
La primera propiedad dice que el producto de potencias de igual base es otra potencia
de la misma base, cuyo exponente es la suma de los exponentes dados.
La segunda se refiere a una potencia de otra potencia: el resultado es otra potencia de
la misma base, cuyo esponente es el producto de los exponentes dados.
La tercera expresa que el cociente de potencias de igual base es otra potencia de la
misma base, cuyo exponente es la diferencia entre el exponente del dividendo y el
exponente del divisor.
La cuarta y quinta propiedad se refieren a que la potenciacin es distributiva con
respecto al producto y con respecto al cociente.

1. (a) 4x4 2x2 = 22 x4 2x2 = 23 x6


36a3 b4 22 32 a3 b4
(b) = = 2 3 ab3 = 6ab3
2
6a b 2 3 a2 b
(c) (2x3 )4 = 214 x34 = 24 x12 = 16x12
2.5. LOS NMEROS REALES 25

Se extiende la definicin para el caso de exponente cero y exponente negativo. Para ello
se define:
a0 = 1 (a = 0)
y
1
an = (n > 0 y a = 0)
an
Con estas definiciones las leyes de los exponentes continan vigentes y la restriccin en
la propiedad 3 (de que m > n) ya no hace falta.

2.5.6 Radicacin de nmeros reales


Dado un nmero real a, se define como la raz ensima de a al nmero real b, siempre que b
elevado a la ensima potencia de por resultado a. En smbolos:
n
a = b bn = a

donde a y b son nmeros reales no negativos y n es un entero positivo mayor que 1, o a y b


son nmeros reales negativos y n un entero positivo impar.
El nmero a es el radicando, el smbolo es el signo radical, n es el ndice y n a es la
raz n-sima de a.

Ejemplo 32 3 27 = 3 y 5
32 = 2.

Si el radicando es positivo, cuando el ndice es par se tienen dos soluciones y, si no se dice


lo contrario, se considera slo la solucin positiva, que se la llama raz ensima principal.

5
Ejemplo 33 25 =
5

Por convencin se considera 25 = 5 (recuerda que si el ndice se omite escribir, el ndice
es 2).
Cuando el ndice es par y el radicando es negativo, el resultado no es un nmero real.

Ejemplo 34 49 / R.

2.5.7 Propiedades de la radicacin


Si n es un entero positivo distinto de 1 y a y b nmeros reales, tales que las races existan,
se tiene que:

1. ( n a) = a

2. n a b = n a n b

a n
a
3. n = (b =
 0)
b n
b
26 CAPTULO 2. LOS NMEROS REALES

4. m n
a = mn a

5. n a = ak , con k Z+
n.k


6. Si n es impar (n = 1), n an = a

a si a 0
7. Si n es par, a =
n n
a si a < 0

Ejemplo 35 Indica las propiedades usadas.



(a) 5 32x10 y 5 = 2x2 y

(b) 3
64 = 6 64 = 2

2.5.8 Exponentes racionales


Si b es un nmero real y n un entero positivo, se define:
1
n
bn = b

siempre que la raz exista.



Ejemplo 36 91/2 = 9 = 3 y 321/5 = 5
32 = 2.

Si n y m son enteros positivos (n = 1) sin divisores comunes, excepto 1 o 1, definimos:


m
 m
n n
bn = b = bm

siempre que esta expresin sea un nmero real, esto es, si b 0 cuando n es par o bien, si
n es impar.
Finalmente definimos:
m 1
b n = m/n (b = 0)
b
Se puede trabajar tanto con races como con exponentes racionales y las propiedades de
la potenciacin siguen siendo vlidas.

2.5.9 Racionalizacin de denominadores


Racionalizar el denominador de una fraccin significa transformar la fraccin dada en otra
equivalente en cuyo denominador no figure ningn signo radical.
Muchas operaciones que comprenden radicales se facilitan si al principio se racionalizan
todos los denominadores.

Ejemplo 37 Racionalizar los denominadores de las fracciones dadas.



2 2 2 2 2 2 2
(a) = =  2 = = 2
2 2 2 2 2
2.6. TRABAJO PRCTICO 27
   
5 5 2+ 3 5 2+ 3
(b) =     =  2  2 =
2 3 2 3 2+ 3 2 3
 
5 2+ 3  
= = 5 2 + 3
23

2 6
Ejercicio 2.5.1 Cmo se racionaliza el denominador de ?
2+ 3 5

2.6 Trabajo Prctico


1. Completa la tabla, calculando el valor numrico de las expresiones simblicas:
a b c a + (c + c) b + (c) c + (a b) (a + c) (a c)
4 1 0
6 5 4
0 7 7
3 6 3
1 3 1
2. Ubica en la recta real los siguientes nmeros:

(a) 5 (b) 3 (c) 8


13 (d) - 13
4

3. Completa los resultados de las tablas siguientes. Incluye todos los clculos auxiliares.
19 19
+ 3 3
5 5
6 2 12 6 2 12
17
2 1 5
24 1
1 1
0, 3 0, 3

4. Sean los siguientes nmeros:


(a) 1, 75 (b) 0, 3333... (c) 25
5 (d) 134
99 (e) 0, 2777... (f) 11
6
i. Exprsalos en forma decimal o fraccionaria, segn corresponda.
ii. Ubcalos en la recta numrica.

5. Decide si las siguientes afirmaciones son verdaderas (V) o falsas (F).

5
(a) es un elemento del conjunto R.
6
(b) 10 pertenece a N.

(c) 3 es un nmero irracional.
(d) Todo nmero entero es un nmero real.
28 CAPTULO 2. LOS NMEROS REALES

(e) Existen nmeros naturales que no son enteros.



(f) 16 es un nmero irracional.
(g) R pero
/ Q.

6. Completa la tabla marcando con una cruz su pertenencia a los conjuntos indicados:
  
8 8
36 e 3 27 4, 1 2 0, 123 2 3 8 0, 101001000... 4
N
Z
Q
I
R

7. Resuelve utilizando propiedades de la potenciacin y la radicacin:


1 2 1 1
(a) 3 5 3 5 3 5 3 5
(2u)3 (2u)5
(b) +
(2u)5 (2u)3
 3  6  13
9 3 4 3
(c)
16 4 3 4

(d) 4 22 24

(e) 4a2 b2

(f) 256x4 y 2

(g)
7
5a28 b14 c49

8. Utiliza notacin cientfica para escribir los nmeros dados.


(a) 1.270.000.000.000 (b) 0, 000000000000341 (c) 0, 0080000000321

9. Utiliza notacin decimal para escribir los nmeros dados.


(a) 9, 87 1017 (b) 4, 02 104 (c) 1, 00423 1014

10. Calcula el resultado.


 
1 1 1 1
(a) 1 + + +
2 3 4 5
   
12 1 1 1 1
(b) + 12+
24 9 5 4 5
1 2 1
+
(c) 9 6 1 4
1+ 7
2.6. TRABAJO PRCTICO 29
 
1 1        2 
(d) + 1
6 1 15 + 2 19 + 1
18 3 25 14 1
3 7 + 3
14
2 3
 2 3 1 1 
(e) 4 + 3
4 12 5 + 58 15 2 2 + 2 (4)
 2 2  1 3    3 5  1 2 4
(f) 3
4 (8) 3 + 2 (10) 25 12 10 1
3 (2) + 6 (1) (3)+ 25 6 3
1 3 7  4  2 2 10    2
(g) 2 4 + 8 3 + (5) 5 5 3 2 + 12 + 32 5
2

 4    1
(h)
3 3
3 21 16 + 8 2
 2   1 3  1 3
(i) 3 8 12 + 4 (2) 12 1
 
      1
(j) (5) 2 18 12 (4)2 (2)3 12

11. En el criadero de perros La Mascota Feliz venden anualmente 150 animales. Por
cada cachorro se invierten $25 en productos veterinarios y, adems, cada animalito
consume 6 kg de alimento balanceado mientras est en el lugar. El criador compra el
alimento en bolsas de 20 kg que cuesta $63 cada una. Si cada perrito se vende a $110
Cul es la ganancia anual del criadero?
12. Claudio tiene depositado sus ahorros en tres bancos distintos. En el banco Seguro,
guarda la cuarta parte de los que tiene en el banco Alcanca, donde coloc $2.892, y
en el banco Confianza deposit una suma igual a la diferencia entre los dos anteriores.
Antes de irse de viaje, decidi distribuir sus ahorros en parte iguales en los tres bancos.

(a) Qu movimiento de dinero entre las cuentas de los tres bancos tiene que hacer?
(b) Cunto dinero tiene ahorrado?

13. En una carrera infantil de bicicletas y triciclos hay 7 pilotos y 19 ruedas Cuntos
triciclos participan?
14. De cuntas maneras puede expresarse 15 como suma de nmeros naturales consecu-
tivos?
15. En una embotelladora de jugos se necesitan fraccionar jugos de manzana y pera, sin
mezclarlos. Tienen 54 litros de jugo de manzana y 42 litros de jugo de pera; y desean
repartirlos en depsitos que contengan un nmero entero de litros:

(a) Cul es la capacidad que tendr el depsito ms grande, comn a los dos tipos
de jugo?
(b) Cuntos barriles de cada jugo se obtienen?

16. En un pueblo, el 14% de sus habitantes es menor de 18 aos. Si la diferencia de los de


ms edad con respecto a los menores de 18 es de 2.880 personas Cuntos habitantes
tiene el pueblo?
30 CAPTULO 2. LOS NMEROS REALES

17. Tres de cada 5 habitantes de una ciudad son menores de edad, y 6 de cada 11 menores
son varones. Si en la ciudad hay 7.200 varones menores de edad Cuntos habitantes
hay en la ciudad?
18. En un grupo de ratones de laboratorio,75% son de pelaje oscuro y el resto, de pelaje
blanco. Entre stos, el 50% tiene ojos azules; y entre los de pelaje oscuro, el 20%. Si
en total hay 99 ratones con ojos azules, cul es el nmero total de ratones?
1
19. De las 4 horas que Julin dispone como tiempo libre, utiliza del tiempo jugando en
5
1 3
la casa, leyendo, viendo televisin, y el resto jugando en la calle. Cunto tiempo
4 8
dedica a esta ltima actividad?
20. En un campo rectangular de 15 hm de ancho y 2, 5 km de largo, los guaranes sembraron
el 40% de mandioca y maz, las dos terceras partes del resto la sembraron con zapallos
y el resto lo destinaron para vivir ellos con los suyos.

(a) Cuntas hectreas sembraron con mandioca y maz?


(b) Qu porcentaje de tierra destinaron para vivr ellos?
(c) Cuntas hectreas represent lo sembrado con zapallo?

21. De un grupo de 522 aborgenes, la sexta parte se dedicaban a recolectar frutos y de


los restantes, la mitad ms uno vivin de la caza y la pesca. Los dems consideraban
que los recursos eran escasos y decidieron trasladarse a otro lugar.

(a) Cul es el nmero de aborgenes que se dedicaban a recolectar frutos?


(b) Cuntos aborgenes decidieron marcharse a otro lugar?
(c) Cuntos se dedicaban a la caza y pesca?
Captulo 3

Polinomios

3.1 Definicin
Se llama polinomio en una variable a una expresin de la forma:

an xn + an1 xn1 + . . . + a2 x2 + a1 x + a0 , n N0

a0 , a1 , a2 , ., an son nmeros reales y se denominan coeficientes del polinomio y x es la variable


o indeterminada. Si el coeficiente an = 0 se dice que el polinomio tiene grado n. Cada
expresin de la forma ak xk es un trmino del polinomio y el coeficiente de la potencia ms
alta del polinomio es el coeficiente principal.
Si todos los coeficientes de un polinomio son ceros, entonces el polinomio se llama poli-
nomio nulo y se denota con 0.

3.2 Grado
Se llama grado de un polinomio al mayor nmero entero positivo al que est elevada la
variable. El grado de un polinomio p(x) se abrevia con grad [p(x)].

Ejemplo 38

(a) Si p(x) = x3 x7 +3x9 , los coeficientes son a0 = 0, a1 = 0, a2 = 0, a3 = 1, a4 = 0,


a5 = 0, a6 = 0, a7 = 1, a8 = 0 y a9 = 3; el grado es 9 y la indeterminada es x.
(b) q(x) = 2y 6 + 5y 4 4y 7; los coeficientes son a0 = 7, a1 = 4, a2 = 0, a3 = 0,
a4 = 5, a5 = 0 y a6 = 2, el grado es 6 y la indeterminada es y.
(c) r(x) = 2 es un polinomio de grado cero, donde a0 = 2 y la indeterminada es x.
(d) s(x) = 0 es un polinomio en la variable x, donde todos los coeficientes son cero y
se dice que no tiene grado. Se lo conoce con el nombre de polinomio nulo.

31
32 CAPTULO 3. POLINOMIOS

3.3 Valor numrico


Es el valor que se obtiene al sustituir la indeterminada por el valor dado y luego efectuar
las operaciones.

Ejemplo 39 Dado p(x) = x3 + 3x2 + 5, halle el valor numrico de p en x = 1 y en x = 2.

Solucin
p(1) = 13 + 3 12 + 5 = 9
p(2) = (2)3 + 3 (2)2 + 5 = 8 + 3 4 + 5 = 8 + 12 + 5 = 9

3.4 Igualdad
Se dice que dos polinomios son iguales si tienen el mismo grado y son iguales los coeficientes
de los trminos de igual grado.

3.5 Polinomios opuestos


Dado p(x) = a0 + a1 x + a2 x2 + ... + an xn , con n N, el polinomio opuesto es el que se
obtiene cambiando los signos a cada uno de los coeficientes del polinomio. Se denota con
p(x).

Ejemplo 40 Si q(x) = 2x4 3x3 + 4x2 1, entonces el opuesto de q es:

q(x) = 2x4 + 3x3 4x2 + 1

3.6 Operaciones
3.6.1 Suma
La suma de dos polinomios es otro polinomio. Para sumar polinomios se suman entre s los
coeficientes de los trminos semejantes, es decir de los que tienen igual grado.

Ejemplo 41 Si p(x) = 3x3 + 5x2 x + 1 y q(x) = 2x3 2x2 + 7x 2, halla p(x) + q(x).

Solucin
Una regla prctica consiste en escribir en distintas filas a los polinomios que se quiere
sumar, encolumnando los trminos semejantes o de igual grado (aquellos en los que la
incgnita aparece elevada a la misma potencia):

p(x) = 3x3 + 5x2 x + 1


+
q(x) = 2x3 2x2 + 7x 2

p(x) + q(x) = 5x3 + 3x2 + 6x 1


3.6. OPERACIONES 33

Es decir que:
p(x) + q(x) = 5x3 + 3x2 + 6x 1
Ejemplo 42 Encuentra p(x) + q(x) siendo p(x) = 2x5 3x4 4x3 + 5x2 7x + 6 y q(x) =
x4 + 5x2 + 8x 2.
Solucin
p(x) = 2x5 3x4 4x3 + 5x2 7x + 6
+
q(x) = x4 + 5x2 + 8x 2

p(x) + q(x) = 2x5 4x4 4x3 + 10x2 + x + 4


Luego resulta:
p(x) + q(x) = 2x5 4x4 4x3 + 10x2 + x + 4
Ejemplo 43 Determina p(x) + q(x), con p(x) = 4x4 + 12 x3 3x + 5 y
q(x) = 4x4 32 x3 + 6x2 + 3x 5.
Solucin
1 3
p(x) = 4x4 + 2x 3x + 5
+
3 3
q(x) = 4x4 2x + 6x2 + 3x 5

2 3
p(x) + q(x) = 2x + 6x2
Finalmente:
p(x) + q(x) = x3 + 6x2
Para pensar: cmo es el grado de la suma con respecto al grado de los sumandos?

3.6.2 Resta
El resultado es otro polinomio, que se obtiene sumando al minuendo el opuesto del sustraen-
do. En smbolos:
p(x) q(x) = p(x) + [q(x)]
Ejemplo 44 Dados p(x) = 5x3 + 2x2 + 1 y q(x) = 9x3 2x2 + 4x 5, obtiene p(x) q(x).
Solucin
p(x) = 5x3 + 2x2 + 1
+
q(x) = 9x3 + 2x2 4x + 5

p(x) q(x) = 4x3 + 4x2 4x + 6


Luego, p(x) q(x) = 4x + 4x 4x + 6.
3 2
34 CAPTULO 3. POLINOMIOS

3.6.3 Producto
El producto de dos polinomios es otro polinomio. Para encontrarlo, se aplica la propiedad
distributiva y luego se suman los trminos de igual grado.

Ejemplo 45 Dados p(x) = 2x3 + 5x2 3 y q(x) = 3x 1, obtiene p(x) q(x).

Solucin
p(x) q(x) = (2x3 + 5x2 3) (3x 1)

= 2x3 3x + 5x2 3x 3 3x + 2x3 (1) + 5x2 (1) + (3)(1)

= 6x4 + 15x3 9x 2x3 5x2 + 3

= 6x4 + 13x3 5x2 9x + 3

A veces, por comodidad se los ubica de la siguiente manera:

2x3 + 5x2 3
3x 1
6x4 + 15x3 9x
3 2
2x 5x +3
6x4 + 13x3 5x2 9x + 3

Para pensar: cmo es el grado del polinomio producto con respecto al grado de los
factores?

3.6.4 Divisin
Algoritmo de la divisin
Dados dos polinomios p(x) y q(x), q(x) = 0, siempre es posible encontrar dos nicos poli-
nomios c(x) y r(x), llamados polinomios cociente y resto, respectivamente, donde r(x) = 0
o grado de r(x) <grado de q(x), tales que se verifica que el dividendo es igual al producto
del divisor por el cociente ms el resto. Esto es:

p(x) = q(x) c(x) + r(x), con r(x) = 0 grad [r(x)] < grad [q(x)]

Recuerda que para dividir dos polinomios se ordena el dividendo y el divisor de acuerdo a
las potencias decrecientes de la variable. El procedimiento para dividir dos polinomios con-
siste en tomar el primer trmino del dividendo y dividirlo por el primer trmino del divisor,
obtenindose as el primer trmino del cociente. Este trmino se multiplica seguidamente
por el divisor y se resta el producto obtenido del dividendo, encontrndose el primer resto
parcial. Se repite el mtodo descripto hasta que el resto sea cero o de grado menor al del
divisor.
3.6. OPERACIONES 35

Ejemplo 46 Dados p(x) = 10x2 + 7x 10 y q(x) = 2x + 3, halla el polinomio cociente y el


polinomio resto.

Solucin
10x2 + 7x 10 2x + 3
10x2 15x 5x 4
8x 10
+8x + 12
+2
Luego, c(x) = 5x 4 y r(x) = 2.
Definicin Si en una divisin entre los polinomios p(x) y q(x), resulta el resto r(x) = 0,
se dice que el polinomio p(x) es divisible por q(x) o bien que q(x) es un divisor o factor de
p(x).

Divisin sinttica
La divisin sinttica se realiza cuando el divisor es un binomio de la forma x a, donde a
es un nmero real.
Mediante este mtodo se obtienen los coeficientes del polinomio cociente y del polinomio
resto, que es de grado cero o es cero. Por qu?
Veamos cul es el procedimiento para realizar la divisin sinttica:

1. El polinomio dividendo y el polinomio divisor (que es de grado uno) deben estar


completos y ordenados en forma decreciente.
2. El coeficiente principal del cociente es igual al coeficiente principal del dividendo.
3. Cada uno de los sucesivos coeficientes se obtienen multiplicando el ltimo coeficiente
del cociente por el nmero a y sumando este resultado al prximo coeficiente del
dividendo.
4. El ltimo coeficiente que se obtenga, corresponde al resto de la divisin.

La disposicin prctica la veremos con un ejemplo.

Ejemplo 47 Usa divisin sinttica para encontrar p(x) : q(x).

(a) p(x) = x4 7x2 + 8x 42 y q(x) = x 3


Solucin
1 0 7 8 42

3 3 9 6 42
1 3 2 14 0
En este caso resulta c(x) = x3 + 3x2 + 2x + 14 y r(x) = 0. Como r(x) = 0 se dice
que p(x) es divisible por q(x).
Se observa que el polinomio cociente es de grado menor en uno que el polinomio
dividendo.
36 CAPTULO 3. POLINOMIOS

(b) p(x) = x3 x2 + 3x 1 y q(x) = x + 2


Solucin
1 1 3 1

2 2 6 18
1 3 9 19
En este caso c(x) = x2 3x + 9 y r(x) = 19.

Teorema del resto


El resto de la divisin de un polinomio en x por un binomio de la forma x a, es el valor
numrico del polinomio dividendo para x = a. En smbolos:

p(x) = (x a) c(x) + r r = p(a)

Ejemplo 48 Sea p(x) = x3 x2 + 3x 1 y q(x) = x + 2. Halla directamente el resto de la


divisin entre p(x) y q(x).

Solucin
Para hallar directamente el resto de la divisin entre p(x) y q(x), se debe hallar el valor
del polinomio dividendo en x = 2:

p(2) = (2)3 (2)2 + 3 (2) 1 = 8 4 6 1 = 19 = r

Luego r = p(2) = 19 como se obtuvo anteriormente.

3.6.5 Raz de un polinomio


El nmero a se llama raz de un polinomio p(x), si el valor numrico de p(x) en x = a es
cero:
a es raz de p(x) si p(a) = 0

Ejemplo 49 Sea p(x) = 6x3 5x2 7x + 4. Es 1 una raz de p(x)?

Solucin
Hallemos el valor numrico de p para x = 1:

p(1) = 6(1)3 5(1)2 7(1) + 4 = 6 5 + 7 + 4 = 0.

Como p(1) = 0 se concluye que 1 es una raz de p(x).

3.6.6 Orden de multiplicidad de una raz


Si tomamos el polinomio p(x) = x2 2x + 1 = (x 1)2 = (x 1)(x 1), se observa que el
factor x1 aparece dos veces. Se dice en este caso que 1 es una raz doble o de multiplicidad
dos. En general, si x a aparece como factor de un polinomio k veces, diremos que a es una
raz de multiplicidad k del polinomio.
3.7. FACTORIZACIN DE POLINOMIOS 37

3.6.7 Teorema del factor


x a es un factor de p(x) si p(a) = 0.

Ejemplo 50 Sea p(x) = x3 3x2 + x 3 Es x 3 un factor de p(x)?

Solucin
Hallemos el valor de p(3):

p(3) = 33 3 32 + 3 3 = 27 27 + 3 3 = 0

Se concluye que x 3 un factor de p(x) (3 es una raz de p(x)).

3.7 Factorizacin de polinomios


Factorizar un polinomio significa encontrar una expresin equivalente que sea el producto
de dos o ms polinomios.

Ejemplo 51 Sabiendo que 4 es una raz de p(x) = 2x3 11x2 + 10x + 8, se pide factorizar
el polinomio.

Solucin
Se aplica el teorema del factor. Se puede ir disminuyendo el grado del polinomio divi-
dendo, mediante la divisin sinttica (regla de Ruffini):

2 11 10 8

4 8 12 8
2 3 2 0

Luego c(x) = 2x2 3x 2 y r(x) = 0 (el resto deba ser cero, como sucedi, ya que 4 es
una raz de p(x)).
Al polinomio dividendo se lo puede expresar (recuerda el algoritmo de la divisin de
polinomios) como el producto del cociente por el divisor:

p(x) = (2x2 3x 2)(x 4)

Y factorizando 2x2 3x 2 (sugerencia: halla las races de la ecuacin cuadrtica


2x 3x 2 = 0 mediante la frmula cuadrtica) se llega a que:
2

 
2 1
p(x) = (2x 3x 2)(x 4) = 2 x + (x 2)(x 4)
2

3.7.1 Casos de factoreo


Recordaremos los distintos casos de factoreo.
38 CAPTULO 3. POLINOMIOS

Primer caso: factor comn

Es un factor que figura en todos los trminos de una suma.

Ejemplo 52

(a) Sea p(x) = 5x + 10x2 + 25x4


Observamos que el factor 5x figura en todos los trminos del polinomio, por lo que
haciendo uso de la propiedad distributiva del producto con respecto a la suma,
se tiene:
p(x) = 5x + 10x2 + 25x4 = 5x(1 + 2x + 5x3 )
(b) Sea q(x) = 2(x + 2)2 + 8(x + 2)
Observamos que el factor 2(x + 2) figura en los dos trminos de la suma, por lo
que puede ser extrado como factor comn:
q(x) = 2(x + 2)2 + 8(x + 2) = 2(x + 2) [x + 2 + 4] = 2(x + 2)(x + 6)

Segundo caso: factorizacin por grupos

Se puede utilizar la propiedad distributiva para factorizar polinomios con un nmero par de
trminos.

Ejemplo 53

(a) Factoriza p(x) = 6x3 9x2 + 4x 6.


Solucin
Puede escribirse:
6x3 9x2 + 4x 6 = (6x3 9x2 ) + (4x 6)

= 3x2 (2x 3) + 2(2x 3)

= (2x 3)(3x2 + 2)
(b) Factoriza q(x) = x3 + x2 x 1.
Solucin
En este caso resulta:
x3 + x2 x 1 = (x3 + x2 ) (x + 1)

= x2 (x + 1) (x + 1)

= (x + 1)(x2 1)
3.7. FACTORIZACIN DE POLINOMIOS 39

Tercer caso: trinomio cuadrado


Dentro de los trinomios de segundo grado estn aquellos que son cuadrados perfectos y los
que no lo son.
Ejemplo 54 Factoriza los polinomios dados.
(a) p(x) = 9x2 + 30x + 25 = (3x + 5)2
(b) p(x) = 2x2 + 10x 12 = 2(x2 + 5x 6) = 2(x 1)(x + 6)
En el primer ejemplo se tiene que dos de los trminos son cuadrados perfectos y
que el tercer trmino es el doble producto de la raz cuadrada de los otros dos.
(Trinomio cuadrado perfecto)
El segundo ejemplo no es un trinomio cuadrado pefecto.
Ejemplo 55 Veamos un ejemplo que combine dos casos. Factoriza p(x) = 3x3 3x2 6x.
Solucin
Observamos que el factor 3x figura en todos los trminos del polinomio, por lo que
haciendo uso de la propiedad distributiva del producto con respecto a la suma, se tiene:

p(x) = 3x3 3x2 6x = 3x(x2 x 2)

Ahora bien, el polinomio x2 x 2 se factoriza como (x 2)(x + 1). Luego resulta:

p(x) = 3x3 3x2 6x = 3x(x2 x 2) = 3x(x 2)(x + 1)

Cuarto caso: cuatrinomio cubo perfecto


Recordemos que el cubo de un binomio se escribe como:

(x + a)3 = x3 + 3x2 a + 3xa2 + a3


Ejemplo 56 Factoriza p(x) = x3 + 6x2 + 12x + 8
Solucin
El primero y el cuarto trminos son, respectivamente, el cubo de x y de 2, mientras que
los trminos restantes son -alternadamente- el triplo del cuadrado de una de las bases por
la otra. Luego, resulta:

x3 + 6x2 + 12x + 8 = (x + 2)3

Quinto caso: diferencia de cuadrados


Recordemos que una diferencia de cuadrados se expresa como:

x2 a2 = (x a)(x + a)
Ejemplo 57 Factoriza los polinomios dados.
(a) x2 9 = x2 32 = (x 3)(x + 3)
(b) x4 16 = (x2 )2 42 = (x2 4)(x2 + 4) = (x 2)(x + 2)(x2 + 4)
40 CAPTULO 3. POLINOMIOS

Sexto caso: suma o diferencia de potencias de igual grado


Se presentan los siguientes casos:

(a) n par
i. El polinomio xn + an no es divisible ni por x + a ni por x a.
ii. El polinomio xn an es divisible por x + a y por x a.
(b) n impar
i. El polinomio xn + an es divisible por x + a.
ii. El polinomio xn an es divisible por x a.

Ejemplo 58 Factoriza cuando sea posible.

(a) x3 + 8 = x3 + 23 = (x + 2)(x2 2x + 4) (el segundo factor se obtiene al realizar


la divisin sinttica).
(b) x5 32 = x5 25 = (x 2)(x4 + 2x3 + 4x2 + 8x + 16)
(c) x4 1 = (x2 )2 1 = (x2 1)(x2 + 1) = (x 1)(x + 1)(x2 + 1)
(d) x6 + 64 = x6 + 26 no es divisible por x + 2 ni por x 2. Puede factorizarse de
otro modo?
(e) x2 9 = (x 3)(x + 3)

3.8 Trabajo Prctico


1. Decide si las siguientes expresiones algebraicas son polinomios o no. En caso afirmativo,
indique el grado y el trmino independiente.

(a) x4 3x5 + 2x2 + 5



(b) x + 7x2 + 2

(c) 2 x4
2
(d) 2 x 7
x
(e) x3 + x5 + x2
(f) x 2x3 + 8
(g) x3 x 7
2

2. D los valores de a, b, c para que los polinomios dados sean iguales.

(a) p(x) = ax2 + bx + 5; q(x) = 3x2 4x + c


(b) p(x) = 5x3 + ax2 bx ; q(x) = 2x5 cx + 5x3
(c) p(x) = 3 7x4 + 2x2 + bx; q(x) = ax4 + 2x2 + cx
3.8. TRABAJO PRCTICO 41

3. Escribe

(a) Un polinomio ordenado sin trmino independiente.


(b) Un polinomio no ordenado y completo.
(c) Un polinomio completo sin trmino independiente.
(d) Un polinomio de grado 4, completo y con coeficientes impares.

4. Escribe un polinomio que cumpla con las condiciones indicadas en cada caso.

(a) Un polinomio de grado 7, de tal forma que el trmino independiente sea igual a
2, est ordenado en forma creciente, completo y que cada coeficiente sea el doble
que el anterior.
(b) Un polinomio de grado 5, ordenado en forma decreciente, con coeficiente principal
1.

5. Escribe dos polinomios p(x) y q(x), ambos de grado 3, tales que el grado de p(x)+q(x)
sea:
(a) 3 (b) 2 (c) 1 (d) 0
6. Dados los polinomios:
p(x) = 4x3 2x2 + x 1; q(x) = 3x3 + 4x2 x 2; s(x) = x3 x2 1
encuentra:

(a) p(x) q(x) + s(x)


(b) p(x) + q(x) 2s(x)
(c) q(x) s(x)

7. En los siguientes casos encuentra el cociente c(x) y el resto r(x) de dividir p(x) por
q(x):

(a) p(x) = x3 + x2 14x + 6 q(x) = x 3


1
(b) p(x) = 6x3 2x2 + 9x + q(x) = 3x2 2x + 2
2
(c) p(x) = 0, 001x3 + 0, 15x2 + 7, 5x + 125 q(x) = 5 + 0, 1x
(d) p(x) = x3 + 8 q(x) = x2 2x + 4
(e) p(x) = x3 + 8 q(x) = x + 2

8. Encuentra el resto sin efectuar la divisin, cuando se divide p(x) entre q(x)

(a) p(x) = x2 5x + 2 q(x) = x + 1


(b) p(x) = m 1
4
q(x) = m 1
(c) p(x) = 2x 9x + 3x2 + 7x 12
4 3
q(x) = x 4
42 CAPTULO 3. POLINOMIOS

(d) p(x) = 5x3 3x2 4x 3 q(x) = (x 3)


(e) p(x) = 23x 28 + x 3
q(x) = x + 4
1 3 7 17 1
(f) p(x) = x + x + 0, 3 x2 q(x) = x
2 5 4 2
9. Decide, sin efectuar la operacin, cules de las divisiones siguientes son exactas y
justifica por qu.
 
(a) x5 + 32 (x + 2)
 
(b) x7 + 1 (x + 1)
 
(c) 16 x4 (2 + x)
 
(d) x3 + 1 (x 1)

10. Halla a y b para que el polinomio x5 ax + b sea divisible por x2 4.


11. Encuentra el valor de k para que al dividir 2x2 kx + 2 por (x 2) d resto 4.
12. Determina el valor de m para que 3x2 + mx + 4 admita x = 1 como una de sus races.
13. Halla un polinomio de cuarto grado que sea divisible por x2 4 y se anule para x = 3
y x = 5.
14. Calcula el valor de a para que el polinomio x3 ax + 8 tenga la raz x = 2, y calcula
las otras races.
15. Factoriza en el conjunto de los nmeros reales a los polinomios siguientes:

(a) x2 + 10x + 25
(b) x2 + 6x + 9
(c) x3 + 15x2 + 75x + 125
(d) x2 9
(e) 16x2 36
4 4 1 2
(f) x x
49 9
(g) 1 x2
(h) x4 2x3 + x2
3
(i) 3x2 + 3x +
4
(j) x x x + 1
3 2

(k) x4 4x2 5x3 + 5x + 4


Captulo 4

Ecuaciones

4.1 Las ecuaciones


4.1.1 Introduccin
Una ecuacin es una igualdad entre dos expresiones algebraicas, que contienen nmeros y
letras:
Expresion 1 = Expresi on 2
En general si A(x) y B(x) son expresiones algebraicas en x, la igualdad A(x) = B(x) es
una ecuacin algebraica en la variable x.
Por convencin se usan las ltimas letras del abecedario (u, v, w, x, y, z, ...) para
representar cantidades variables (o incgnitas); los nmeros y en ocasiones las primeras
letras del alfabeto (a, b, c, d, ...) representan magnitudes constantes. Las variables y las
constantes estn combinadas entre si mediante las operaciones de suma, resta, multiplicacin,
divisin, potenciacin y radicacin. La igualdad:
bx 1
2ax3 = x y
a + 3c 2
es una ecuacin, donde las variables x e y estn combinadas -en este caso- con las constantes
2a, b, a, 3c y 12 mediante las operaciones mencionadas.
Claro que en este curso veremos ecuaciones mucho ms sencillas: las ecuaciones lineales,
las ecuaciones cuadrticas y tambin los sistemas de ecuaciones.
Antes de ello, analizaremos brevemente qu valores puede tener la variable en una ecua-
cin.

4.1.2 Dominio
Usaremos la expresin dominio de una ecuacin para designar al conjunto de valores nu-
mricos que puede asumir la variable en la ecuacin.

Ejemplo 59

43
44 CAPTULO 4. ECUACIONES

(a) En la ecuacin 2x + 1 = 3 el dominio es el conjunto de todos los nmeros reales


(R), ya que las operaciones presentes en la ecuacin -la suma y el producto- estn
definidas para todo nmero real, por lo que x puede ser cualquier real.
2 3
(b) En la ecuacin + = 4(x + 3) en cambio, sumamos y multiplicamos,
x1 x+2
pero tambin dividimos. Como la divisin por cero no tiene sentido, debemos
excluir del dominio a los nmeros que anulan a los denominadores presentes, esto
es a 1 y a 2. Luego el dominio puede representarse como:

D = R {1, 2}

4.1.3 Conjunto solucin


La ecuacin 2x + 1 = 3 se verifica para x = 1. Se dice que x = 1 es solucin de la ecuacin
dada. La tarea de resolver una ecuacin consiste en encontrar aqullos valores de la variable
x que hacen verdadera la igualdad. A estos valores los llamamos soluciones de la ecuacin.
El conjunto solucin de una ecuacin es el formado por todas las soluciones posibles. Se
denota con CS o bien S. En el ejemplo dado CS = {1}

4.2 La ecuacin lineal


Problema: Seis amigos desean ir al cine y la entrada cuesta $ 7, 50. Uno de ellos ofrece
poner los $ 20 con que cuenta e invita a que los otros cinco completen lo que falta, en forma
equitativa. Cunto debe aportar cada uno de los cinco amigos invitados?
Solucin
Podemos representar este problema mediante la ecuacin:

5x + 20 = 6 7, 50 (1)

en la que el primer miembro representa el dinero que deben aportar los seis amigos y el
segundo miembro el costo de las seis entradas que desean adquirir. La variable x representa
la cantidad de dinero que debe aportar cada uno de los cinco amigos y el nmero 20 lo
que pone el sexto. Como x representa la cantidad de dinero a aportar por cada uno, x no
puede ser negativa; luego el dominio de esta ecuacin es el conjunto de los nmeros reales
no negativos (Q+ {0}). Si realizamos la multiplicacin indicada, resulta:

5x + 20 = 45 (1)

Luego, 5x debe ser igual a 25 y, por tanto, x = 5. Es decir, cada uno de los cinco amigos
invitados debe aportar $ 5.

Definicin Una ecuacin lineal con una incgnita es aquella que puede escribirse en
b
la forma ax + b = 0, con a y b nmeros cualesquiera y a = 0. Su solucin es x = y
a
se obtiene restando b de ambos miembros y dividiendo ambos lados por a. La (1) es una
ecuacin lineal.
4.2. LA ECUACIN LINEAL 45

4.2.1 Ecuaciones equivalentes


Una ecuacin es equivalente a otra si tiene la misma solucin. Para buscar la solucin de una
ecuacin lineal la transformaremos en una ecuacin equivalente a la dada pero ms simple
que sta; con la nueva ecuacin procederemos de igual modo y as sucesivamente, hasta que
en la ecuacin obtenida la solucin sea evidente.

4.2.2 Operaciones permitidas


Obtendremos ecuaciones equivalentes siempre que:

1. Sumemos -o restemos- en ambos lados de la ecuacin el mismo nmero (o la misma


expresin).(por la propiedad uniforme de la suma)
2. Multipliquemos -o dividamos- ambos lados por el mismo nmero (o la misma expresin)
distinto de cero. (por la propiedad uniforme del producto)

La forma de resolver una ecuacin es usar las operaciones permitidas para obtener ecua-
ciones equivalentes a la dada, hasta obtener una ecuacin donde la incognita aparezca sola
en un miembro. La idea es llevar a un miembro todos los trminos que contengan la variable
y en el otro miembro todos los trminos que no contengan la variable (trminos indepen-
dientes). Debe quedarle claro que el hecho de pasar de un miembro al otro equivale a usar
la propiedad uniforme de la igualdad.

4.2.3 Ejemplos
Ejemplo 60 La ecuacin 3x 5 = 7 es una ecuacin lineal. Halla su solucin.

Solucin
Partimos de la ecuacin dada:

3x 5 = 7 (1)

Sumamos 5 a ambos lados: (propiedad uniforme de la suma)

3x 5 + 5 = 7 + 5 (2)

Operamos:
3x = 12 (3)
Dividimos ambos lados por 3: (propiedad uniforme del producto)

x=4 (4)

Las ecuaciones (2), (3) y (4) son equivalentes a la dada (1) y en la ltima (4) la solucin,
x = 4, es evidente.
Por lo tanto el conjunto solucin es CS = {4}

Ejemplo 61 Encuentra la solucin de la ecuacin 2x 7 = 3 3x.


46 CAPTULO 4. ECUACIONES

Solucin
2x 7 = 3 3x (es la ecuacin dada)

2x 7 + (7 + 3x) = 3 3x + (7 + 3x) (sumo 7 + 3x)

5x = 10 (asocio y opero algebraicamente)

x = 2 (divido por 5)

CS = {2}

Observemos que usamos la expresin sumo 7 + 3x en lugar de la ms clara sumo 7 + 3x


a ambos miembros, por economa en el uso del lenguaje. Por igual razn decimos divido
por 5.

Ejemplo 62 Martn acaba de cumplir 55 aos y ya le est fallando la memoria. Tiene dos
hijas, Carla y Andrea, pero no recuerda sus edades. Su mujer, impaciente, le dice: Andrea
es 2 aos mayor que Carla y, dentro de 5 aos, la suma de sus edades ser igual a la mitad
de la que tu tengas entonces (si an existes). Pudo Martn, con la ayuda de su encantadora
mujer, recordar las edades de sus hijas? Cuntos aos tienen estas?

Solucin
Si llamamos x a la edad actual de Carla, la hija menor, Andrea tiene actualmente x + 2
aos, ya que le lleva dos aos a su hermana. Dentro de 5 aos, entonces, Carla tendr x + 5
aos, Andrea x + 7 aos y Martn 60. Luego, la ecuacin que describe la situacin planteada
por la amorosa esposa de Martn es la siguiente:

60 2 = (x + 5) + (x + 7)

O bien:
30 = 2x + 12
De donde:
x=9
Luego, Carla tiene 9 aos, Andrea 11 y Martn queda muy agradecido a su media naranja.

4.3 La ecuacin cuadrtica


Problema: La suma de las cifras correspondientes a los dgitos de un nmero de dos
dgitos es 9. El nmero correspondiente al dgito de las decenas es tres unidades mayor que
el cuadrado del nmero correspondiente al dgito de las unidades. Halla el nmero original.
Solucin
Sabemos que un nmero de dos cifras se puede escribir como xy, donde x es la cifra de
las decenas e y es la cifra de las unidades.
4.3. LA ECUACIN CUADRTICA 47

La suma de las cifras correspondientes a los dgitos de un nmero de dos dgitos es 9,


esto es:
x+y =9 (1)
El nmero correspondiente al dgito de las decenas es tres unidades mayor que el cuadrado
del nmero correspondiente al dgito de las unidades. Es decir: x = 3 + y 2 (2)
Reemplazando (2) en (1) obtenemos: 3 + y 2 + y = 9. O sea 3 + y2 + y 9 = 0
y2 + y 6 = 0
Lo que queda formada es una ecuacin cuadrtica y enseguida veremos cules son sus
soluciones.
Llamamos ecuacin cuadrtica a una expresin de la forma:

ax2 + bx + c = 0,

donde a, b y c son nmeros reales cualesquiera, pero a = 0.


Por ejemplo, x2 3x + 2 = 0 es una ecuacin cuadrtica o ecuacin de segundo grado.
Aunque ms adelante veremos que existen otras alternativas, por ahora usaremos, para
encontrar la solucin de una ecuacin cuadrtica, una frmula, conocida como la frmula
cuadrtica:
b b2 4ac
x= (1)
2a
La demostracin de esta frmula no la veremos en este curso. Se ver en las asignaturas
Introduccin a la Matemtica, Matemtica para Informtica y Matemtica 1 de la Facultad
de Ciencias Exactas.

4.3.1 Ejemplos
Ejemplo 63 En la ecuacin dada como ejemplo, x2 3x + 2 = 0, resulta a = 1, b = 3 y
c = 2.

Sustituyendo estos valores en la frmula (1), resulta:


3+1

= 4
=2

2 2
(3) (3)2 4 1 2 3 98 3 1 31
x= = = = =
21 2 2 2



31 = 2
=1
2 2

Las soluciones o races de esta ecuacin son x = 2 y x = 1, lo que tambin puede indicarse,
usando la notacin de conjuntos, como S = {1, 2}. Observemos que, en este caso, las
soluciones son los nmeros reales y distintos 1 y 2.

Ejemplo 64 Resuelve la ecuacin 3x2 + 6x 3 = 0.

Solucin
48 CAPTULO 4. ECUACIONES

En este caso a = 3, b = 6 y c = 3; sustituyendo en (1) queda:



6 + 0 6

= =1

6 6
6 62 4(3)(3) 6 36 36 6 0 6 0
x= = = = =
2(3) 6 6 6


6 0

= 6
=1
6 6

A diferencia del ejemplo anterior, en este caso hay una sola raz, que es el nmero real x = 1.
Diremos que la ecuacin cuadrtica dada tiene una raz doble (el 1). El conjunto solucin
es CS = {1} o bien S = {1}.

Ejemplo 65 Encuentra las races de la ecuacin 2x2 + x = 54

Solucin
Si reescribimos la ecuacin dada, igualada a cero, resulta: 2x2 + x + 54 = 0. Aqu, a = 2,
b = 1 y c = 54 . Sustituyendo en la frmula, obtenemos:

1 12 4 2 5
4 1 1 10 1 9
x= = =
22 4 4

Como 9 no es un nmero real, decimos que esta ecuacin cuadrtica tiene soluciones no
reales; ms adelante veremos que estas races son nmeros complejos, no reales. En este
caso el conjunto solucin en R es S = .
Para el ejemplo dado, las soluciones obtenidas pueden escribirse, con una notacin usada
en los nmeros complejos, como: x = 14 + 34 i y x = 14 34 i. Cuando estudiemos el
universo de los nmeros complejos llamaremos a estos nmeros complejos y conjugados.
Por ahora diremos que, cuando el radicando es un nmero negativo, la ecuacin cua-
drtica tiene soluciones no reales.

4.3.2 Naturaleza de las races


Antes vimos que si el radicando era cero la ecuacin cuadrtica tena una raz doble, y si el
radicando era positivo las races eran nmeros reales diferentes.
Observamos entonces que, la naturaleza de las races, es decir , el tipo de races que se
presentan -reales y distintas, reales e iguales (doble) o complejas y conjugadas- depende
del radicando, esto es del nmero que aparece bajo el signo radical.
Debido a su importancia, este nmero tiene un nombre propio y un smbolo particular: se
simboliza con la letra griega delta mayscula ( = b2 4ac) y se denomina el discriminante
de la ecuacin cuadrtica.
b + b2 4ac
Si > 0 las races son reales y distintas y estn dadas por x1 = y
2a
b b 4ac
2
x2 = .
2a
4.3. LA ECUACIN CUADRTICA 49
 
b b
Si = 0 las races son reales e iguales y estn dadas por x12 = por lo que CS =
2a 2a
Si < 0 las races no son reales y el conjunto solucin es CS =

Ejemplo 66 El discriminante de la ecuacin x2 +4x+3 = 0 es = 42 413 = 1612 = 4


y esto significa que la ecuacin tiene dos races reales y distintas.

Ejemplo 67 En un tringulo issceles, la base excede a la altura en 20 cm. Cul es la


longitud de cada lado si el rea del tringulo mide 150 cm2 ?

Solucin
Ilustremos este problema, dibujando el tringulo en cuestin:

El dato es el rea (A) que, como sabemos, se calcula con la frmula:

base altura
A=
2
En nuestro caso, si se llama x a la altura, la base es b = x + 20

(x + 20) x
A= = 150
2
Luego:
x2 + 20x 300 = 0
Aplicando la frmula, con a = 1, b = 20 y c = 300, resulta: x = 10 y x = 30. Como
la longitud debe ser un nmero real positivo, elegimos x = 10 y no tomamos en cuenta la
raz negativa; en este caso, la naturaleza del problema limita los posibles valores para x -el
dominio- a los nmeros reales positivos (Podra ser x = 0? Por qu?). Luego la altura es
de 10 cm y la base mide 30 cm.
Debemos calcular ahora los lados iguales; si llamamos a a cada uno de ellos y aplicamos
el teorema de Pitgoras en cualquiera de los dos tringulos rectngulos formados, resulta:
 2
2 x + 20
x + = a2
2
50 CAPTULO 4. ECUACIONES

x + 20
ya que las dos partes en que queda dividida la base son iguales y miden, por lo tanto,
2
(puedes dar la razn de sto?)
O sea:
(x + 20)2 x2 + 2 x 20 + 202
x2 + = x2 + = a2
4 4
Sustituyendo x por el valor encontrado de 10 cm y operando, obtenemos:
100 + 400 + 400 900 1.300
a2 = 100 + = 100 + = = 325 cm2
4 4 4
Luego:
a= 325 cm2  18, 0277 cm
por lo que cada lado uno de los lados iguales del tringulo mide, aproximadamente, 18 cm.
Solucin al problema inicial
La ecuacin planteada es y 2 + y 6 = 0. Se sabe que a = 1, b = 1 y c = 6. Resolviendo
se obtiene:
1 + 5 4

= =2

2 2
1 12 4 1 (6) 1 1 + 24 1 25 1 5
y= = = = =
21 2 2 2



1 5 = 6 = 3
2 2
Los valores de la variable obtenidos son y = 2; y = 3 y como la ecuacin (2) es x = 3 + y 2 ,
reemplazando el valor de y por y = 2 se obtiene el valor de x.

x = 3 + 22 x = 7

Como el nmero buscado es xy, reemplazando x e y por los valores obtenidos, se tiene que
el nmero es 72.
Si ahora en (2) reemplazamos y por 3 se tiene que x = 3 + (3)2 x = 12, por lo
que esta solucin se descarta, ya que, recordemos que x es la cifra de las decenas y no puede
tener dos dgitos.

4.3.3 Propiedades de las races


Entre los coeficientes a, b y c de la ecuacin cuadrtica ax2 + bx + c = 0 (a = 0) y sus races
x1 y x2 , existen las relaciones siguientes:
b c
x1 + x2 = (1) y x1 x2 = (2)
a a
Prueba
b b2 4ac b b2 4ac
x1 = + x2 =
2a 2a 2a 2a
4.3. LA ECUACIN CUADRTICA 51

Sumando miembro a miembro ambas expresiones, resulta:

b b b b 2b b
x1 + x2 = = = =
2a 2a 2a 2a a

como afirmamos en (1).


Si ahora multiplicamos x1 por x!2 , obtenemos: !

b b2 4ac b b2 4ac
x1 x2 = +
2a 2a 2a 2a
!2
 2
b b2 4ac
=
2a 2a

b2 b2 4ac
= 2

4a 4a2
b2 b2 + 4ac
=
4a2
4ac
=
4a2
c
=
a
como decimos en (2).

Ejemplo 68 Encuentre una ecuacin cuadrtica que tenga por races a 2 y 1/2.

Solucin
Las relaciones encontradas anteriormente nos permiten escribir que

1 b 1 c
2 + = y 2 =
2 a 2 a

3
Es decir, b = a y c = a. Como los valores de b y c estn dados en trminos de a, para
2
cada a que elijamos obtendremos un par de valores de b y c. Por ejemplo, si fijamos a = 1,
3
resultarn b = y c = 1, con lo que la ecuacin pedida ser:
2
3
x2 + x 1 = 0
2

Como la nica limitacin para a es no valer cero, habr infinitas ecuaciones que cumplan
con la condicin dada en el enunciado (una para cada a elegido).
52 CAPTULO 4. ECUACIONES

4.3.4 Factorizacin
Las expresiones obtenidas en la seccin anterior nos permiten asegurar que la ecuacin de
segundo grado puede factorizarse como se indica:
ax2 + bx + c = a(x x1 )(x x2 )
Prueba
En efecto, si operamos en el segundo miembro de esta igualdad, resulta:
ax2 + bx + c = a(x x1 )(x x2 )

= (ax ax1 )(x x2 )

= ax2 ax2 x ax1 x + ax1 x2

= ax2 (ax1 x + ax2 x) + ax1 x2

= ax2 ax(x1 + x2 ) + ax1 x2

b c
= ax2 ax( ) + a
a a

= ax2 + bx + c
como queramos probar.

4.3.5 Ecuaciones de Primer Grado con dos incgnitas


Consideremos por ejemplo ecuaciones del tipo 2x + 5y = 3 que son ecuaciones lineales con
dos incgnitas. Cualquier par de nmeros (x, y) que satisfaga a la ecuacin es una solucin
de la misma.
Si en la ecuacin dada, despejamos y en funcin de x, obtenemos:
3 2
y= x
5 5
Asignando valores a x, se obtiene los correspondientes valores de y. Por ejemplo:
x=5 y = 75

x=4 y = 1

3
x=3 y=
5
1
x=2 y=
5
1
x=1 y=
5
4.4. SISTEMAS DE ECUACIONES 53

Se pueden obtener tantas soluciones como se quiera con solo asignar valores a x para
obtener los correspondientes valores de y. Las soluciones son pares ordenados de nmeros
reales. En el ejemplo dado, entre las muchas soluciones que tiene la ecuacin podemos
mencionar las siguientes:
7 3 1 1
(5, ); (4, 1); (3, ); (2, ); (1, )
5 5 5 5
Una ecuacin como la dada es del tipo ax + by = c donde a, b, c son nmeros reales y se
llama ecuacin lineal o de primer grado con dos variables.
La representacin grfica de la ecuacin ax + by = c es una recta.

4.4 Sistemas de ecuaciones


Consideremos ahora dos ecuaciones lineales con dos incgnitas, como por ejemplo:

3x 4y = 11
(1)
2x + 3y = 4

Las soluciones consisten en pares ordenados (x, y) de nmeros que satisfacen ambas
ecuaciones simultneamente. Podemos comprobar que (1, 2) es una solucin para el sistema
dado. Este sistema es llamado sistema lineal porque las incgnitas aparecen elevadas a la
potencia uno en ambas ecuaciones. Para resolver un sistema debemos encontrar los posibles
valores (x, y) que hagan verdadera las dos ecuaciones. Llamamos conjunto solucin de un
sistema al conjunto formado por todos los pares que son solucin del sistema, esto es, que
satisfacen sus ecuaciones simultneamente. En el ejemplo dado, el conjunto solucin es
{(1, 2)}.
Los sistemas de ecuaciones pueden resolverse por distintos mtodos. stos consisten en
aplicar distintas herramientas que son operaciones que nos permiten ir del sistema dado a
otro equivalente.
Se dice que dos sistemas son equivalentes si tienen las mismas soluciones. Por ejemplo:
 
x+y =5 3x y = 11
2x + y = 9 6x 2y = 22
no son equivalentes, ya que la solucin del primero (nica) es el par (4, 1), mientras que
el segundo sistema adems del par (4, 1) tiene otras soluciones.
Las operaciones que tenemos permitido usar son:
Operacin 1 : intercambiar las posiciones de dos ecuaciones.
Operacin 2 : multiplicar una ecuacin por una constante no nula, esto es, reemplazar una
ecuacin por otra que es mltiplo no nulo de ella.
Operacin 3: sumar un mltiplo de una ecuacin a otra, esto es, reemplazar una ecuacin
por la suma de ella y el mltiplo de otra.
Veremos ahora los distintos mtodos de resolucin de sistemas.
Mtodo de sustitucin: Se despeja una incgnita de una de las ecuaciones y se
reemplaza el valor de esta incgnita en la otra ecuacin. Se resuelve para la otra incgnita.
54 CAPTULO 4. ECUACIONES

Mtodo de igualacin: Se despejan una misma incgnita de ambas ecuaciones y se


igualan los valores obtenidos. De esta manera se tiene una ecuacin lineal en la otra incgnita
que se resuelve.
Mtodo de adicin por sumas y restas: A partir del sistema dado, se trata de
obtener, otro que tenga el mismo coeficiente de una de las incgnitas en ambas ecuaciones
y de signos opuestos. Se suman los miembros correpondientes de las ecuaciones obtenidas y
se resuelve, para la incgnita que queda, la ecuacin resultante.
Aqu estn los detalles para la resolucin del sistema (1) por el mtodo de sustitucin.

Ejemplo 69 Resolver el sistema (1) anterior.

Solucin
4y + 11
Resolviendo la primera ecuacin para x, obtenemos x = (1). Cuando sustitumos
3
esta expresin en la segunda ecuacin y operamos, resulta:
4y + 11
2 + 3y = 4
3
2(4y + 11) + 9y = 12
8y + 22 + 9y = 12
17y = 34
y = 2

Sustituyendo este valor de y en (1), resulta:

4(2) + 11 8 + 11
x= = =1
3 3
Luego x = 1 e y = 2 y el conjunto solucin es CS = {(1, 2)}.
Este sistema tiene solucin nica.

Ejemplo 70 Resolver el sistema:

2x 2y = 4
3x + 5y = 14

Solucin

Usando la operacin 2, reemplazamos al dado por el sistema:

6x + 6y = 12
6x + 10y = 28

Entonces, usando la operacin 3,

16y = 16
y = 1
4.4. SISTEMAS DE ECUACIONES 55

En esta forma la solucin es obvia. La segunda ecuacin dice que y = 1 y, cuando este
valor es sustitudo en la primera, del sistema dado resulta que x = 3. Verificamos que el par
ordenado (3, 1) satisface a ambas ecuaciones del sistema original. Luego, (3, 1) es la nica
solucin. El conjunto solucin es CS = {(3, 1)}

Ejemplo 71 Resolver el sistema:

xy = 2
3x 3y = 5

Solucin

Usando la operacin 2, multiplicamos a la primera ecuacin por 3. A la segunda la


dejamos como est:

3x + 3y = 6
3x 3y = 5

Por el mtodo de sumas, sumamos miembro a miembro a las ecuaciones del ltimo sistema
y obtenemos:
0 = 11

que es un absurdo. Luego CS =

Ejemplo 72 Resolver el sistema:

xy = 1
2x 2y = 2

Usando la operacin 2, multiplicamos a la primera ecuacin por 2. A la segunda la


dejamos como est:

2x + 2y = 2
2x 2y = 2

Por el mtodo de sumas, sumamos miembro a miembro a las ecuaciones del ltimo
sistema y obtenemos:

0=0

que es una identidad, pero no est la incgnita. Esto significa que el sistema tiene infinitas
soluciones. Las soluciones son todos los puntos que se encuentran sobre la recta de ecuacin
x y = 1. Luego conjunto solucin es CS = {(x, y) | x y = 1}
56 CAPTULO 4. ECUACIONES

4.4.1 Interpretacin grfica

En la seccin anterior encontramos nuestro primer sistema de ecuaciones. Descubrimos que


este sistema tena al par ordenado (1, 2) como solucin. As, x = 1 e y = 2 hace que
ambas igualdades sean verdaderas. Interpretando grficamente, podemos decir que cada una
de las ecuaciones lineales representa una recta y la interseccin de las dos rectas es, en este
caso, un punto.
En forma general, y desde el punto de vista geomtrico, dado un sistema de dos ecuaciones
lineales con dos incgnitas puede ocurrir que las rectas se corten en un punto, que no se
corten o que sean coincidentes. Teniendo en cuenta las soluciones del sistema se dice que
el mismo tiene solucin nica, en el primer caso. En el segundo caso el sistema no tiene
solucin y en el tercero el sistema tiene infinitas soluciones.
La figura siguiente ilustra estos tres casos.
4.5. TRABAJO PRCTICO 57

Si un sistema tiene solucin nica se dice que es compatible determinado, si tiene in-
finitas soluciones se dice que es compatible indeterminado y si no tiene solucin se llama
incompatible.
Observe que al ser el sistema compatible indeterminado las rectas son coincidentes y se
cortan en los infinitos puntos que esta posee. (por eso se dice que tiene infinitas soluciones).

Ejercicio 4.4.1 Si el sistema x+2y = 4 y ax+3y = b tienen infinitas soluciones diferentes,


cules son a y b?

4.5 Trabajo Prctico


1. Une con flechas, segn corresponda.
La diferencia entre la raz cbica de un nmero y tres (m + n)2
El doble de un nmero ms su siguiente x
(x 1) = 1
La diferencia entre la raz cbica de un nmero y el triple de dicho nmero
3
t3
El cuadrado de la suma de dos nmeros 3
t3t
La suma de los cuadrados de dos nmeros 2 x + (x + 1)
La diferencia entre un nmero y su anterior es uno a2 + b2
2. Escribe en lenguaje algebraico cada uno de los siguientes enunciados.

(a) Un nmero natural, su anterior y su posterior.


(b) La quinta parte del triple de un nmero.
(c) Un nmero menos su mitad ms su doble.
(d) La quinta parte del triple de un nmero.
(e) La diferencia de dos nmeros consecutivos elevados al cuadrado.
58 CAPTULO 4. ECUACIONES

3. Escribe en lenguaje algebraico cada uno de los siguientes enunciados.

(a) El cuadrado de la suma de dos nmeros reales es igual a la suma de sus cuadrados
ms el doble de su producto.
(b) El espacio recorrido por un mvil es igual al producto de su velocidad por el
tiempo que est en movimiento.
(c) El producto de dos potencias de igual base es igual a otra potencia que tiene la
misma base y cuyo exponente es igual a la suma de los exponentes de las potencias
que se multiplican.
(d) Considerando un rebao de x ovejas:
i. Nmero de patas del rebao.
ii. Nmero de patas si se mueren 6 ovejas.
iii. Nmero de ovejas despus de nacer 18 corderillos.
iv. Nmero de ovejas despus de dos aos si el rebao crece un cuarto al ao.

4. Resuelve las siguientes ecuaciones.


y
(a) 4 x 14 4 = 8 (b) 102 + 8 = 12x 62 (c) 12 + 32 = 25
7

(d) 5x 14 2x = 7 4 + 3x (e) 2z + 4 5 + 6z = 52
2 
(f) 6x + 15 2x = 5 4 + 23 (g) 3x + 1 = 3x 5
6 6
5. Determina para qu valor de k el nmero a dado es solucin de la ecuacin.

(a) 2k + 3x 7 = 5 + 2x 3k a = 3
1 5
(b) 4 x + k = k + 3 2x a=
2 2
6. Para cada caso, plantea la ecuacin y resuelva:

(a) Si a la mitad de un nmero se lo disminuye en seis unidades, y al resultado se lo


multiplica por tres, da nueve. Cul es el nmero?
(b) Si al doble de un nmero se lo aumenta en cinco unidaes, y al resultado se lo
divide por tres, se obtiene siete. Halla dicho nmero.
(c) Si a un nmero se le suma su tercera parte y a este resultado se lo resta el mismo
nmero aumentado en 5, se obtiene 1 Cul es dicho nmero?
4
(d) Cul es el nmero cuya tercera parte aumentada en del mismo, pero dismi-
5
nuida en 5 unidades, sobrepasa en 15 al valor del nmero dado?
(e) La suma de tres nmeros pares consecutivos es 102 Cules son los nmeros?

7. Propone problemas que puedan resolverse mediante las siguientes ecuaciones


(a) 3n + 2 = 26 (b) 200 5n = 40 n (c) (2a 1) a = 5
4.5. TRABAJO PRCTICO 59

8. Resuelve las siguientes ecuaciones cuadrticas:


(a) x2 81 = 0 (b) 14x2 + 28 = 0 (c) (x + 6)(x 6) = 13

(d) (2x 5)(2x + 5) 119 = 0 (e) x2 = 7x (f) x2 + 12x + 35 = 0

x2 x
(g) x2 3x + 2 = 0 (h) x2 + 4x = 285 (i) + =2
4 2
1
9. Cul debe ser el valor de c para que la ecuacin, x2 + 2x + c = 0, tenga una raz
5
doble?
10. Resuelve los siguientes problemas.

(a) Si al triple de un nmero se le suma su cuadrado se obtiene 88. Cul es el


nmero?
(b) Un ganadero compra corderos por $12000. Se le mueren tres y el resto los vende
a $3000 ms cada uno de lo que le costo, perdiendo $15000. Cuntos compr y
a qu precio?
(c) Halla dos nmeros consecutivos cuyo producto es 56.
(d) Determina los lados de un rectngulo, sabiendo que su semipermetro es 25 m y
su rea es 150m2 .
(e) La edad de Nelly era hace 6 aos la raz cuadrada de la edad que tendr dentro
de 6 aos. Determina la edad actual.
(f) Halla dos nmeros naturales consecutivos cuyo producto sea igual al cociente
11
entre el cuadrado del mayor y .
10
13
(g) La suma entre un nmero y su recproco es . De qu nmero se trata?
6
(h) La diagonal de un rectngulo tiene una longitud de 13 cm; si la altura es 7 cm
mayor que la base, cul es la superficie del rectngulo?
(i) Si el permetro de un rectngulo es 26cm y su superficie es 40 cm2 , halla la
longitud de su diagonal.

y =x+1
(a) Determina si (1, 2) es una solucin del sistema:
2x + y = 4

y + x = 1
(b) Determina si (3, 2) es una solucin del sistema:
y + 3x = 4

11. Resuelve analtica y grficamente cada uno de los sistemas de ecuaciones:



x+y =6
(a)
xy =2

x+y =8
(b)
2x y = 1
60 CAPTULO 4. ECUACIONES

3x 2y = 5
(c)
7x + 4y = 7
&
5x + 3y = 7
(d) 3 3 37
x y =
2 4 4
12. Resuelve los siguientes problemas:

(a) El permetro de un terreno rectangular es de 36 km. Si el largo del terreno tiene


8 km ms que el ancho, hallar las dimensiones del terreno.
(b) La suma de dos nmeros es 27 y la diferencia entre el mayor y el menor es 3.
Cules son esos nmeros?
(c) Encontrar dos nmeros cuya suma sea 58 y cuya diferencia sea 16.
(d) La diferencia de dos nmeros es 16. Tres veces el ms grande es siete veces el
ms pequeo. Cules son los nmeros?
(e) La suma del largo y el ancho de un rectangulo es de 19 cm. El largo es dos unidades
menor que el doble del ancho. Encontrar el ancho y el largo del rectngulo.
(f) Dos ngulos son complementarios. Su diferencia es 34o . Encontar las medidas de
los ngulos. (ngulos complementarios son aquellos cuya suma es de 90o )
(g) Ramn vende autos y camiones. Tiene espacio en su lote para 510 vehculos.
Por experiencia, sabe que sus utilidades son mayores si tiene 190 autos ms que
camiones. Cuntos vehculos de cada tipo debe tener para obtener la mayor
ganancia?
(h) Una familia acamp en un lugar a 45 km del pueblo. Ellos condujeron 13 km
ms de lo que caminaron para llegar al lugar del campamento. Qu distancia
caminaron?
(i) Marisa es 21 aos ms grande que Laura. En seis aos, Marisa tendr el doble
de la edad de Laura. Qu edad tiene ahora?
Captulo 5

Desigualdades

5.1 Introduccin
Las relaciones de orden fueron introducidas en una seccin anterior. Expresiones tales como
3x 2 < 0 y 6 2x 4 son llamadas desigualdades. Ahora aprenderemos a resolver
desigualdades. Resolver una desigualdad significa determinar los valores de x que hacen ver-
dadera la desigualdad. Para hacerlo la reducimos usualmente a una desigualdad equivalente
2
de una forma ms simple, tpicamente de forma tal como x < o 3 x 2. Estas ltimas
3
desigualdades determinan intervalos sobre la recta real.

Un crculo vaco indica que el punto no est incluido, mientras que un crculo relleno
indica que si lo est. A diferencia de una ecuacin en x, donde el conjunto solucin es
usualmente un nmero o un conjunto pequeo de nmeros, el conjunto solucin para una
inecuacin en x es, por lo general, un intervalo de nmeros (o tal vez una unin de esos
intervalos). Pero as como para resolver ecuaciones, para resolver inecuaciones hay mtodos
algebraicos y geomtricos.
Antes de estudiar la forma de encontrar la solucin de una inecuacin, precisaremos la
idea de intervalo.

61
62 CAPTULO 5. DESIGUALDADES

5.2 Intervalos
Sean a y b dos nmeros reales tales que a < b. Llamamos intervalo a un subconjunto de la
recta real, segn las siguientes definiciones:

5.2.1 Intervalo abierto


Es el conjunto de los nmeros reales comprendidos entre a y b. Lo representamos entre
parntesis, con el nmero menor a la izquierda:

(a, b) = {x R | a < x < b}

5.2.2 Intervalo cerrado


Es el conjunto formado por los nmeros reales a y b y los comprendidos entre ellos. Lo
representamos entre corchetes, con el nmero menor a la izquierda:

[a, b] = {x R | a x b}

La longitud del intervalo [a, b] es el nmero positivo b a.

5.2.3 Intervalo semiabierto (o semicerrado)


Las definiciones anteriores se pueden generalizar considerando la semirrecta y la recta como
intervalos no acotados, lo que se expresa utilizando los smbolos + y . Debemos recor-
dar que estos smbolos se usan solamente por conveniencia de notacin y que no representan
a nmeros reales.
Definimos como intervalo semiabierto a la derecha, de extremos a y b, al conjunto formado
por b y los nmeros reales comprendidos entre a y b. Lo representamos entre un parntesis
y un corchete, con el nmero menor a la izquierda:

[a, b) = {x| x R a x < b}

Anlogamente, el intervalo semiabierto a la izquierda est dado por:

(a, b] = {x|x R a < x b}


5.2. INTERVALOS 63

Generalizamos estas definiciones de la siguiente manera:

(b, ) = {x |x R x > b}

[a, ) = {x |x R x a}

(, d) = {x |x R x < d}

(, c] = {x |x R x c}

(, ) = {x |x R} = R
64 CAPTULO 5. DESIGUALDADES

5.3 Desigualdades lineales


Llamaremos desigualdad lineal en una variable a cualquiera de las siguientes expresiones:
ax + b > 0; ax + b < 0; ax + b 0 o ax + b 0
El smbolo se lee mayor o igual que, as como significa menor o igual que. Por
ejemplo, 8 5 ya que 8 es mayor que 5 (se cumple la relacin de >) y tambin 4 4, ya
que 4 es igual a 4 (en este caso la expresin es verdadera para el signo =).
Como en las ecuaciones lineales, aqu nos interesar especialmente encontrar la solucin
de una desigualdad lineal. A diferencia de la mayora de las ecuaciones, cuyo conjunto
solucin est formado por una cantidad pequea de elementos, los conjuntos de soluciones
de las desigualdades son -en general- infinitos.
El mtodo que seguiremos para encontrar la solucin de una desigualdad consiste -como
en las ecuaciones- en obtener desigualdades equivalentes a la dada, pero cada vez ms simples
que la anterior, hasta obtener una desigualdad que muestre claramente el conjunto solucin
buscado.
Para operar con desigualdades debemos tener en cuenta las siguientes propiedades:
1. Cuando sumamos (o restamos) a ambos lados de la desigualdad un nmero (o una
expresin) cualquiera, la desigualdad se conserva. Por ejemplo:
2<7 dada

2+5<7+5 sumo 5 a ambos lados


(el sentido se conserva)

7 < 12 la desigualdad se mantiene

2. Cuando multiplicamos (o dividimos) ambos lados de una desigualdad por un nmero


(o una expresin) mayor que cero (positiva), la desigualdad se mantiene. Por ejemplo:
5 > 3 dada

5 7 > (3) 7 multiplico ambos lados por 7 > 0


(el sentido se mantiene)

35 > 21 la desigualdad se conserva

3. Cuando multiplicamos (o dividimos) ambos lados de una desigualdad por un nmero (o


una expresin) menor que cero (negativa), la desigualdad cambia de sentido. Ejemplo:
68 dada

6 (5) 8 (5) multiplico ambos lados por 5 < 0


(el sentido cambia)

30 40 la desigualdad cambia de sentido


5.3. DESIGUALDADES LINEALES 65

Ejemplo 73 Encuentra el conjunto solucin de las desigualdades lineales dadas y represn-


talos sobre la recta numrica.
(a) 3x 6 > 0 (b) 4 2x 2 (c) 3x 6 < 4 2x
Solucin
(a)
3x 6 > 0 dada

3x 6 + 6 > 0 + 6 sumo 6

3x > 6 opero

3x : 3 > 6 : 3 divido por 3

x>2 opero
El conjunto solucin est formado por todos los nmeros reales que son mayores
que 2 y puede representarse por: S = {x R | x > 2}. En la recta numrica
cumplen con la condicin dada en la solucin todos los puntos (o nmeros) que
se encuentran a la derecha de 2 (son mayores que 2).

Para indicar que un nmero que se encuentra en un extremo no pertenece a la


solucin -como el 2 en este caso- se conviene en dibujarlo hueco o con un parntesis
sobre l.
(b)
4 2x 2 dada

4 2x 4 2 4 resto 4

2x 2 opero

2x : (2) 2 : (2) divido por 2

x1 opero
66 CAPTULO 5. DESIGUALDADES

El conjunto solucin est formado por todos los nmeros reales que son menores
o iguales que 1 y puede representarse por: S = {x R | x 1}. En la recta
numrica cumplen con la condicin dada en la solucin, el nmero real 1 y todos
los puntos (o nmeros) que se encuentran a la izquierda de 1 (son menores que
1): para indicar que un nmero que se encuentra en un extremo pertenece a la
solucin -como el 1 en este caso- se conviene en dibujarlo lleno o con un corchete
sobre l.
(c)

3x 6 4 2x dada

3x 6 + 6 + 2x 4 2x + 6 + 2x sumo 6 + 2x

5x 10 opero

5x : 5 10 : 5 divido por 5

x2 opero

El conjunto solucin est formado por todos los nmeros reales que son menores
o iguales que 2 y puede representarse por: S = {x R | x 2}. En la recta
numrica cumplen con la condicin dada en la solucin el 2 y todos los puntos (o
nmeros) que se encuentran a la izquierda de 2 (son menores que 2).

Mtodo algebraico para resolver una desigualdad

1. Se trasladan al miembro de la izquieda todos los trminos que contienen la variable y


al miembro de la derecha los trminos constantes.

2. Se realiza la suma de los trminos en cada miembro.

3. Se dividen los dos miembros de la desigualdad obtenida en el paso anterior, entre el


coeficiente de la variable, teniendo cuidado de que si dicho coeficiente es negativo debe
cambiarse el signo de la desigualdad.

5.4 Trabajo Prctico


1. Clasifica cada afirmacin como verdadera o falsa.
1 1
(a) 3 3 (b) 0 (c) 0 (d) 0, 8 > 0, 8
2 4
5.4. TRABAJO PRCTICO 67

2. Determina, en cada caso, si los nmeros dados son solucin de la desigualdad.


(a) x > 4 4; 0; 4; 6;
(b) x 10 4; 10; 0; 11;
(c) y 5 0; 4; 5; 6;
1 2
(d) y 1; ; 0; 0, 5;
2 3
(e) x 0 2; 3; 0; 3;
3. Indica el conjunto solucin de cada una de las siguientes inecuaciones y representa
graficamente.
(a) x3 (b) x5>2 (c) 4x 5 < x

1 2
(d) x<x5 (e) 2x 3 < 5x + 7 (f) x+2 x5
3 3
x+1 x1
(g) (h) 2 (x 3) > x + 5 (i) 2 (x + 1) < 4 (x + 4) 2x
2 3
4. Grafica en la recta real:

(a) Todos los valores de x tales que x < 3 y x > 1


(b) Todos los valores de x tales que x 4 o x 1
(c) Todos los valores de x tales que x 4 y x 1
(d) Todos los valores de x tales que x 2 y x 5

5. Resuelve los siguientes problemas:

(a) Cules son los nmeros de dos cifras tales que al multiplicarlos por 7 el producto
resulta mayor o igual que 658?
(b) Sabiendo que un lado de un tringulo es de 65 cm; otro de 15 cm y el tercer lado
es un nmero exacto de cm que termina en 5. Calcula cul puede ser la longitud
de ese tercer lado.
Sugerencia: Recuerda que en todo tringulo la suma de las medidas de dos lados
es menor que la medida del tercer lado.
(c) Cules son los nmeros naturales impares tales que su triplo, disminuido en 5
unidades, es menor que 46?
(d) Juan, Pedro y Pablo son hermanos. Pablo tiene 11 aos; Juan tiene 5 aos ms
que Pedro, y la suma de los aos de Juan y Pedro no alcanzan a los de Pablo.
Cuntos aos tiene Pedro si su edad es un nmero impar de aos?
(e) La carga mxima de un camin es de 3500 kg. sabiendo que en cada viaje
transporta por lo menos 2800 kg. Cuntos equipos que pesan 70 kg cada uno
puede transportar en cada viaje?
(f) El largo de un rectngulo es 26 cm. Qu ancho debe tener para que el permetro
sea mayor que 80 cm y menor que 100 cm?
68 CAPTULO 5. DESIGUALDADES

(g) La altura de un tringulo es 20 cm. Que longitud en la base da un rea mayor


a 65 cm2 ?
Captulo 6

Funciones

6.1 Introduccin
La nocin de correspondencia se presenta a menudo en la vida diaria. Analicemos algunos
ejemplos:

- A cada persona le corresponde su nmero de documento.

- La presin atmosfrica depende de la altura del lugar en que nos encontramos.

- La longitud L de una circunferencia es una funcin de su radio r (aqu L = 2r).

Los ejemplos dados muestran que en cada una de estas situaciones intervienen dos con-
juntos: un conjunto de partida (el dominio) y un conjunto de llegada (el codominio), a los
que designaremos A y B: personas y nmeros en el primer ejemplo, nmeros para ambos en
los otros dos.
En todos los casos mencionados se pone en evidencia una relacin entre dos magnitudes,
de un modo tal que a cada valor de una de ellas le corresponde un determinado valor de
la otra. En un curso de Matemtica interesa especialmente que A y B sean conjuntos
numricos.
Diremos que una funcin es una regla o procedimiento que asigna a cada nmero x de
un determinado conjunto, un nmero y de otro conjunto. La regla puede ser expresada en
palabras o por una frmula matemtica:

El costo mensual de la energa domiciliaria en Salta se compone de un cargo fijo de $


12, 58, ms $ 0, 0765 por cada kw-h consumido, ms impuestos.

L = 2r (para la longitud de una circunferencia).


e
v = (para la velocidad media de un objeto como funcin del tiempo t, donde e
t
representa el espacio recorrido).

69
70 CAPTULO 6. FUNCIONES

La funcin puede darse tambin por medio de una tabla de datos como es el caso, por
ejemplo, del costo de envo de una encomienda desde Salta a Buenos Aires:

Peso Hasta 1 kg 1 a 2 kg 2 a 3 kg 3 a 4 kg 4 a 5 kg Ms de 5 kg
Costo $ 8, 00 $ 12, 00 $ 16, 00 $ 20, 00 $ 24, 00 $ 24 + 0, 80/kg excedido

6.2 Forma simblica de representar funciones


Describiremos a una funcin con una letra, tal como f , g, h, . . . La frmula particular de
la relacin que define a la funcin, si sta es conocida, se escribe usualmente como y = f (x)
y se lee: y es igual a efe de x o y es una funcin de x. Por ejemplo, la funcin f que
toma algn nmero real y lo eleva al cuadrado, se escribe como:

y = f (x) = x2

En este ejemplo:
f(1) = 12 = 1
   2
1 1 1
f = =
2 2 4

f(3) = (3)2 = 9
Anlogamente, una funcin g que extrae la raz cuadrada de un nmero real no negativo,
puede ser escrita como:
y = g(x) = x
En este caso:

g(25) = 25 = 5

g(100) = 100 = 10
  
1 1 1
g = =
16 16 4

pero, por ejemplo, g(25) no est definido, ya que no es posible en R calcular la raz cuadrada
de un nmero negativo.
Podemos representar una funcin f por medio de un esquema como el siguiente:

f :AB

x  y , donde y = f (x)
y pensar que la funcin f transforma un nmero x de un conjunto A en un nmero asociado
y de un conjunto B. A es el conjunto de partida (dominio) y B es el conjunto de llegada.
Llamamos a x variable independiente y a y variable dependiente, puesto que depende de x.
6.3. DEFINICIN 71

Por ejemplo, la longitud L de una circunferencia dada por la frmula L = 2r, es una
funcin de su radio; en este caso la variable independiente es r y la variable dependiente L.
El espacio recorrido e por un auto en t horas, a una velocidad constante de 90 kilmetros
por hora, est dada por la expresin e = 90t. Aqu t es la variable independiente y e es la
variable dependiente o funcin.
Para cualquier funcin, el conjunto de todos los valores posibles para la variable inde-
pendiente es el dominio de la funcin y el conjunto de todos los valores posibles para la
variable dependiente el rango (o imagen) de la funcin.
Ejemplo 74 Sea la funcin definida por f (x) = x2 ; el dominio es el conjunto de los nmeros
reales (D = R) y la imagen (o rango) es el conjunto de todos los nmeros reales no negativos,
I = R+ 0 , ya que el cuadrado de todo nmero real es mayor o igual a cero.

Ejemplo 75 Para la funcin raz cuadrada, definida por la expresin g(x) = x, el dominio
y el rango son el mismo conjunto, los reales no negativos.

6.3 Definicin
Resumiremos las ideas anteriores en la siguiente definicin:
Una funcin f es una regla y = f (x) que asigna a cada valor posible de la variable
independiente x, un nico valor de la variable dependiente y. El dominio de f es el conjunto
de todos los valores que puede asumir la variable independiente x, mientras que el rango
(tambin imagen) es el conjunto de todos los valores posibles para la variable dependiente
y.
Debemos hacer notar que el rango o imagen no siempre coincide con el conjunto de
llegada o codominio; es decir, rango y codominio son palabras que designan a conjuntos
distintos, aunque stos -a veces- pueden coincidir.

6.4 Funciones lineales


Las funciones ms simples, pero no por eso menos tiles, son las funciones lineales. La
funcin lineal tiene la forma general:
y = f(x) = mx + b
donde m y b son nmeros reales cualesquiera. El nmero m se llama pendiente y b es la
ordenada al origen (el valor de y cuando x es cero), conocida tambin como la intercepcin
con el eje y.
Su representacin grfica es una recta que corta al eje de las x en el valor que corresponde
a la solucin o raz de la ecuacin lineal mx + b = 0. La constante m nos da informacin
sobre la inclinacin de la grfica de la funcin lineal (que es una recta) con respecto a la
horizontal.
Ejemplo 76 Dibuja los grficos asociados con las funciones dadas.
(a) y = f (x) = 2x + 1 (b) y = g(x) = 3x + 2
72 CAPTULO 6. FUNCIONES

Solucin
(a) (b)

Podemos observar que la grfica de la funcin f es una recta que sube, mientras que en
la de la funcin g, la recta baja. Puedes investigar por qu ocurre sto?
Cuando b = 0, la frmula de la funcin lineal se reduce a la expresin y = mx; grfi-
camente corresponde a cualquier recta que pase por el origen, algunas de las cuales se han
dibujado en el grfico:

El costo de cualquier producto que se expende por unidad de medida puede representarse
por una funcin lineal de este tipo. Por ejemplo, si el precio de un kilogramo de azcar es
$ 3, 20; el costo de adquirir x kilogramos est dado por la funcin C(x) = 3, 20x.

6.5 Funciones cuadrticas


Llamamos funcin cuadrtica a la a la funcin f definida por la frmula:

y = f (x) = ax2 + bx + c

donde a, b y c son nmeros reales y a = 0.


6.5. FUNCIONES CUADRTICAS 73

Su grfico es una curva denominada parbola, que puede cortar al eje horizontal en dos
puntos, en uno solo o en ninguno, segn que la ecuacin asociada, ax2 + bx + c = 0, tenga
-respectivamente- dos soluciones reales distintas, una nica solucin real o soluciones no
reales.
As, por ejemplo, la funcin cuadrtica dada por la frmula y = x2 9, corta al eje
horizontal en x = 3 y en x = 3, ya que 3 y 3 son las races de la ecuacin x2 9 = 0. Los
valores de la variable independiente que anulan a la funcin, se denominan los ceros de la
funcin; en este caso, 3 y 3 son los ceros de la funcin cuadrtica dada. Su representacin
grfica es:

El punto ms bajo (o ms alto) alcanzado por la curva en la  representacin grfica



b b2 + 4ac
corresponde al vrtice de la parbola. Este punto tiene coordenadas , . El
2a 4a
b
grfico es simtico respecto de la recta x = . Adems, por su misma naturaleza, toda
2a
parbola tiene un punto de cambio. Si la parbola abre hacia arriba (esto ocurre cuando
a > 0), este punto corresponde al valor mnimo de la funcin. Si abre hacia abajo (cuando
a < 0), al mximo valor de la funcin.
En la figura vemos los grficos asociados con las funciones cuadrticas y = x2 , y = x2 ,
y = x2 + 3, y = x2 4x + 3, y = x2 + 4x + 3 y y = x2 + 4x + 3.
74 CAPTULO 6. FUNCIONES

Observa cmo influyen en la forma de la grfica los valores de los parmetros a, b y c.


Graficamos ahora la fincin f dada por f(x) = x2 5x + 6.
Si hacemos f(x) = 0 obtenemos la ecuacin cuadrtica correspondiente: x2 5x + 6 = 0
Resolviendo se obtiene las races reales x1 = 2 y x2 = 3. Su grfico se muestra en la
figura; notemos que el grfico corta al eje x dos veces, una vez cuando x = 2 y otra cuando
x = 3.

As, vemos que, para cualquier funcin cuadrtica, las races reales de una ecuacin
cuadrtica corresponden a los puntos donde la parbola asociada corta al eje x.
6.6. TRABAJO PRCTICO 75

Dado que las races son 2 y 3 para esta ecuacin cuadrtica, podemos factorizar el
polinomio como
x2 5x + 6 = (x 2)(x 3)
Por consiguiente, vemos que las races reales de una ecuacin cuadrtica corresponden a los
factores lineales del polinomio cuadrtico. As, si sabemos que una parbola corta al eje x
en un punto x = r, entonces x = r es tambin un cero de la funcin cuadrtica asociada y
x r es un factor de la expresin cuadrtica.
Dependiendo de la orientacin de la parbola (hacia arriba o hacia abajo) y la posicin
del punto de cambio, una parbola puede no tocar nunca al eje x, como con x2 + 3 que
consideramos antes. En tal caso, la correspondiente ecuacin cuadrtica tiene, no obstante,
dos races, pero ellas son races no reales y se llaman complejas. A los nmeros complejos los
estudiaremos en las asignaturas Introduccin a la Matemtica, Matemtica para Informtica
o Matemtica 1.
Veamos ahora algunas ideas claves sobre el crecimiento y el decrecimiento de funciones,
cuyas definiciones las veremos ms adelante:
Una funcin f es creciente si los valores de y = f (x) crecen cuando crece x.
Una funcin f es decreciente si los valores de y = f(x) decrecen cuando crece x.
El grfico de una funcin creciente sube cuando nos movemos de izquierda a derecha.
El grfico de una funcin decreciente baja cuando nos movemos de izquierda a derecha.

6.6 Trabajo Prctico


1. Dados los siguientes grficos, indica si alguno de los grficos no corresponde a una
funcin. Justifica.
76 CAPTULO 6. FUNCIONES

2. Dados los siguientes grficos de funciones, determina en cada caso, en qu intervalos


es creciente, en que intervalos es decreciente, en qu punto alcanza su mximo, cul es
dicho valor mximo, en qu punto alcanza su mnimo y cul es su valor mnimo.

3 3

2 2

1 1

-4 -2 00 2x 4 -2 -1 00 1 2
x

-1 -1

3. Pablo va a comprar una empresa creada en 1989. Entre la informacin que le ofrecen,
figura este grfico.

(a) Cul es el perodo de dficit (los gastos superan a los ingresos)? A qu cres
que se debi?

(b) Cul fue el momento de mayor supervit (los ingresos superan a los gastos)?

(c) En qu momento comenz a ser rentable?

(d) En el perodo en que los ingresos permanecen constantes, tambin permanece


constante el beneficio de la empresa? Por qu?

(e) Analiza los valores entre los cules las funciones crecen o decrecen.
6.6. TRABAJO PRCTICO 77

4. Grafica en un sistema de ejes las funciones definidas de R en R cuyas frmulas son:


f (x) = 2x + 1
(a) g(x) = 2x 4

h(x) = 2x

f (x) = 4x + 2
(b) g(x) = 14 x + 2

h(x) = 2x + 2

f (x) = 5x 1
(c) g(x) = 15 x 1

h(x) = 3x 1

Observa las representaciones efectuadas en (a), (b) y (c), y explica cmo crees
que influyen los nmeros b y m en la grfica de una funcin lineal.

5. Cul de las frmulas siguientes es la que corresponde a este grfico? Explic el porqu
de tu eleccin.

(a) y =x+1 (b) y =1x (c) y =x1


78 CAPTULO 6. FUNCIONES

-3 -2 -1 00 1 2 3
x
-1

-2

6. Grafica las siguientes funciones definidas de R en R:


(a) y = 5x + 3
2 (b) y = 2, 5x 4 (c) y = 3
7. Un barril vaco de 30 litros de capacidad pesa 1, 5 kg. Le agregamos agua destilada y
queremos saber cmo calcular la variacin del peso del barril respecto de la cantidad
de agua a medida que se llena.

(a) Deduce la frmula de dicha funcin.


(b) Qu cantidad de agua se habrn agregado si al pesar el barril la balanza indica
10 kg?
(c) Grafica la funcin.
(d) Cul es el dominio de la funcin?

8. Si la medida del lado de un cuadrado aumenta cinco veces:

(a) Cuntas veces aumenta su permetro?


(b) Cuntas veces aumenta su rea?
(c) Encuentra una frmula asociada al permetro y grafica la funcin. Qu tipo de
funcin es?
(d) La funcin asociada al rea, es lineal? Por qu?

(a) Grafica con distintos colores y en un mismo sistema de ejes las funciones cua-
drticas de R en R cuyas frmulas son:
(i) y = x2 (ii) y = 2x2 (iii) y = 4x2 (iv) y = x2

(v) y = 2x2 (vi) y = 0, 5x2 (vii) y = 0, 5x2 (viii) y = 4x2

(b) Cul es el eje de simetra y el vrtice de cada una de las parbolas?


(c) Compara las grficas obtenidas con la grfica de f(x) = x2 . Como crees que
influye el nmero a en la grfica de una funcin cuadrtica?
6.6. TRABAJO PRCTICO 79

9. Indica qu frmula corresponde a cada una de las parbolas de la figura siguiente.


Fundamenta tu respuesta.
(a) y = 3x2 (b) y = 0, 4x2 (c) y = 4x2

-3 -2 -1 00 1 2
x
-2

-4

-6

10. Para cada una de las siguientes funciones cuadrticas:

(a) Determina races, vrtice, eje de simetra de cada una de las parbolas.
(b) Realiza el grfico correspondiente.
(c) A partir del grfico determina: el dominio y la imagen de la funcin.
(d) Indica en qu intervalo crece, en que intervalo decrece, dnde es positiva.
(i) y = x2 x 2 (ii) y = 2x2 + 4x 5
2 (iii) y = 3x2 12x + 12

(iv) y = 12 x2 + 72 x 5 (v) y = 14 x2 32 x + 11
4

11. En una isla se introdujeron 112 iguanas. Al principio se reprodujeron rpidamente,


pero los recursos de la isla comenzaron a escasear y la poblacin decreci. El nmero
de iguanas a los t aos de haberlas dejado en la isla est dado por: i(t) = t2 + 22t +
112 (t 0).

(a) Determina la cantidad de aos en los cuales la poblacin de iguanas aument.


(b) Cul fue la mayor cantidad de iguanas que hubo? En qu momento se alcanz
esa cantidad?
(c) En qu momento la poblacin de iguanas se extingue?

12. Al poner a prueba un nuevo automvil se comprob que para velocidades mayores que
10 km/h y menores que 150 km/h, el rendimiento de nafta r (en km/litro) est rela-
cionado con la velocidad v (en km/h) mediante la funcin: r(v) = 0, 002v (180 v) .
Averigua a qu velocidad el rendimiento es mximo y calcula dicho rendimiento.
80 CAPTULO 6. FUNCIONES

13. Un beb pesa tres kilos y medio al nacer y tres aos despus alcanza diez kilos. Supone
que el peso P (en kg) en la infancia est relacionado linealmente con la edad t (en
aos).

(a) Expresa P en trminos de t.


(b) Cul ser P cuando el nio cumpla 6 aos?
(c) A qu edad pesar 25 kg?
(d) En un plano tP , dibuja una grfica que muestre la relacin entre P y t para
0 t 12.

14. Sean las siguientes funciones cuadrticas:


(a) y = 2x2 8x (b) y = 3x2 18x 15

(c) y = 12 x2 + 2x + 2 (d) y = 3x2 18x + 39


En cada caso:
i. Determina las intersecciones con los ejes y el vrtice de la parbola (suge-
rencia: el vrtice est ubicado entre los ceros de la funcin y equidista de
ellos).
ii. Dibuja la grfica.
iii. A partir de la grfica realizada, determina dominio e imagen.

15. El nmero de kilmetros K que cierto automvil puede recorrer con un litro de nafta,
a una velocidad de v kilmetros por hora, est dado por:
1 2 4
K= v + v para 0 < v < 120
150 5
(a) Representa grficamente la funcin en el plano vK.
(b) Indica la velocidad ms econmica para un viaje.
(c) Determina el valor mximo de K.

Вам также может понравиться